You are on page 1of 426
The Princeton Review: Verbal Workbook for the ® Version 1.0 Copyright © 2015 by TPR Education IP Holdings, LLC. All Rights Reserved. No part of this manual may be reproduced for distribution to a third party in any form or by any means, electronic or mechanical, including photocopying, recording, or any information retrieval system, without the prior consent of the publisher, The Princeton Review. Permission to reprint this material does not, constitute review or endorsement: by the Educa- tional Testing Service, of this publication as a whole or of any other sample questions or test ing information it may contain. ‘This Manual is for the exclusive use of Princeton Review course students, and is not legal for resale. Educational Testing Service and ETS are registered trademarks of the Educational Testing Service, SAT is a registered trademark of the College Board. ‘The Princeton Review is not affiliated with Princeton University or the Educational Testing Service. 866.7PR.PREP/ www-PrincetonReview.com ACKNOWLEDGMENTS ‘The Princeton Review would like to give a very special thanks to the following people for their extraordinary efforts in the production of this title: Vietoria Zoe Gannon, Joelle Cotham, Anne Morrow, Claudia Landgrover, Kathryn Menefee, and Deborah A. Silvestrini, We would alsc like to thank the following people for their tremendous contribu- tions to this workbook: Sean Adkins, Brian Becker, Clarissa Constantine, Joelle Cotham, Lori DesRochers, Michelle Gellman, George Gittinger, Teresa Johnson, Lisa Mayo, Alexander Palmer, Paige Raschke, Adam Redfield, Teresa Schuberg, Rebecca Scott, Stephen Shuck, David Strauss, Alice Swan, and Jess Thomas. —Jonathan Chiu National Content Director of High School Programs ‘The Princeton Review ConrTENTS Reading Homework Passages .. Passage 1: U.S./World Literature Passage. Questions. . Answers and Explanations. . Passage 2: History/Social Studies Passage. Questions, sev Answers and Explanations. Passage 3: Science Passage. Questions....onnnne Answers and Explanations... Passage 4: History/Social Studies Passage. Questions, Answers and Explanation: Passage 5: History/Soci Passage Questions. aes Answers and Explanations, Passage 6: History/Social Studies Passage Questions, Answers and Explanations. 1 Studies: ay u 16 18 19 22, 26 26 30 31 34 errRetrestonP tgs UE | ¥ Passage 7: History/Social Studies Passage aaa eon 36 Questions... — 36 ‘Answers and Explanations. 40 Passage 8: History/Social Studies Passage... en ses 44 Questions. — 44 Answers and Explanations. . 48 Passage 9: Science Passage 50 Questions. 50 Answers and Explanations. o 54 Passage 10: History/Social Studies Passage 56 Questions. 56 ‘Answers and Explanations ? 58 Passage LI: History/Social Studies PASSAgE Sener 60 Questions: : 60 Answers and Explanations... penne 62. Passage 12: History/Social Studies Passage = c 64 Questions... — 64 Answers and Explanations, 66, Passage 13: History/Social Studies Passage... 68. Questions. o : 68. Answers and Explanations... ee 70 Passage 14: History/Social Studies Passage... a = R Questions. aeveeeee . R Answers and Explanations.......nsnne ™ Passage 15: History/Social Studies Passage o ee 8 Questions. we 8 [Answers and Explanations. on 80 Passage 16: History/Social Studies Passage... : a 82. Questions... = penne 82, Answers and Explanations... a 86 Passage 1 Passage . . et 88 Questions. oo . 89 Answers and Explanations. 92 Passage 18: Science (Dual Passage) Passage = se 96 Questions.. eee 97 Answers and Explanations. 100 Wi | ren smain tlds Ue Passage 19; Science (Dual Passage) Passage... 102 Questions, 103 Answers and Explanations. 106 Writing and Language Homework Passages. 109 Passage 1: What Actually is an Actuary? Passage. 110 Questions, 10 Answers anc Explanations. ua Passage 2: In Defense of Public Defenders Passage sen = — 16, Questions. a 115, ‘Answers anc Explanations... 18 Passage 3: Cultural Anthropology for the Market to Be Passage... 120 Questions, 120 Answers an¢ Explanations. 128 Passage 4: Forensic Anthropology Passage snne 125 Questions. 125 Answers an¢ Explanations... 129 Passage 5: Dieticians Passage... 131 Questions. 131 Answers anc Explanations... 134 Passage 6: UN Translator Passage . 136 ‘Questions. 0 136. Answers and Explanations... 139 Passage 7: The Ethies of Photojournalism Parsauc eee woe AD Questions. = MI Answers an¢ Explanations... 144 Passage 8: Epidemiology Passage penne : 46 Questions... 146 Answers and Explanations, M9 Passage 9: Primatologist Passage : 161 Questions. eae “ os 161 Answers and Explanations... = . 164 Passage 10; Transportation Engineers Passage. oes 156 Questions. 166 Answers and Explanations 159 Passage 11: Concert Pianist Passage. : 161 Questions. 161 Answers and Explanations. 164 erase Hae | i Passage 12: Physical Therapy Passage ‘Questions. Answers and Explanations. Passage 13: School Superintendent Passage Questions. Answers and Explanations Passage 14: Behind the Scenes: Biomedical Engineers Passage Questions. Answers and Explanations, Passage 15: Speech Therapists Passage Questions. Answers and Explanations. Passage 16: Bargaining Over the Cuban Embargo Passage Questions. o Answers and Explanations Passage 17: Henry's History Passage Questions... Answers and Explanations, Passage 18: Who Wins Elections? Passage. Questions. Answers and Explanations. Passage 19: Inventor of the Telephone Passage Questions. Answers and Explanations Passage 20: Bag Ban Debate Passage Questions, Answers and Explanations. Passage 21: The Half-Century War Passage . Questions. Answers and Explanations. 7 Passage 22: A Whale of a Tale - The Myth of Christopher Columbus Passage Questions. Answers and Explanations... Passage 23: The Work of Upton Sinclair Passage Questions. Answers and Explanations. 166 166 169 im im a4 176 176 179 181 181 184 186 186 189 191 191 194 196 196 199 201 201 204 206 206 209 2 au 215, Passage 24: Behind Every Good Movement... Passage. Questions, Answers and Explanations... Passage 25: The Legacy of Watergate Passage Questions... Answers and Explanations. “ juffering for Women’s Suffrage Answers and Explanations... Passage 27: Fair Funding in Education PASsAgC ones Questions, Answers and Explanations. Passage 28: The Destructive Past of the Nobel Peace Prize Passage Questions, . Answers and Explanations. Passage 28: We the Elite of the United States Passage, Questions. ‘Answers and Explanations Passage 30: The Misquoted Queen: Marie Antoinette and her Infamous Tagline Passage Questions.. Answers and Explanations. Passage 31: The Rise of Urban Art Passage Questions. Answers and Explanations... Passage 32: The Gift of Reciprocity Passage Questions... Answers and Explanations. Passage 33: The Happiness of Exercise Passage Questions. Answers and Explanations.. Passage 34: Night: The Last Frontier Passage... Questions.. Angwers and Explanations... Passage 35: Jane Austen, Mistress of Game Theory | Passage so Questions, 7 Answers and Explanations. 228 228 281 233 233 236 238 238 241 243 243 246 248 248 251 258 253 256 258 258 261 268 263 266 268 268 2m 273 273 276 278 278 281 283 283 286 ‘otra ctucatonP iting | Passage 36: The Pace of Place Passage Questions... Answers and Explanations. Passage 37: The Compassionism Model Passage. Questions. vo ‘Answers and Explanations, Passage 38: The Rationale of Seared Cows Passage ... Questions. Answers and Explanations, Passage 39: The Changing Birth Rate Passage Questions. Answers and Explanations. Passage 40: The Pros and Cons of Urban Farming Passage Questions. Answers and Explanations... Passage 41: The Increasing Utility of Service Animals Passage Questions... Answers and Explanations. ee Passage 42: Monkey See, Monkey Do Passage Questions. Answers and Explanations... Passage 43: The America-Car Love Affair Passage Questions. Answers and Explanations. Passage 44: How Common are the Tragedies? Passage Questions. Answers and Explanations. Passage 45: The Disappearing Bee Passage Questions. Answers and Explanations. Passage 46: The Science of Self-Talk Passage. Questions. . Answers and Explanations, Passage 47: How Cont: Passage Questions. Answers and Explanations us is Ebola? 288 288 291 293 293 296 298 298 301 303 303 306 308, 308 Bul 318 313 316 318. a8. Ban 323 328 326 828 328 BBL 333 336 838 338 341 343 343 B47 Passage 48: A Plethora of Senses Passage . Questions. ‘Answers and Explanations. Passage 49: Nutritional Poverty Passage Questions. Answers and Explanations, Passage 50: The Evolving View Passage Questions. “ Answers and Explanations. Passage 51: What the Frack is Going On? Passage os Questions... Answers and Bxplanations. Passage 52: The Algae is Always Greener Passage. Questions... Answers and Explanations. Passage 58: The Cilantro Conundrum Passage . ‘Questions. 7 “Answers and Explanations... Passage 54: The Final Mosquito Solution Passage Questions... “Answers ard Explanations Passage 55: Where are all the Solar Panels? Passage Questions. se Answers ani Explanations... Passage 56: When the Polar Vortex Breaks Passage Questions... Answers and Explanations. Passage 8% There May Not Be Life on Mars, But at Least There's Gas Passage... Questions Answers and Explanations. Passage 6f: The Complicated Future of 3D Printing Passage... a Questions. Answers and Explanations 349 B49 358 355 355 358 360 360 363 365 365 368 370 370 373 375 a5 378, 380 380 33 385 885, 388 390 390 393 395 395 398 400 400 403 (oTPaeiestontP lige |X Passage 59: Human Echolocation: Do You See What I See? Passage Questions. Answers and Explanations. Passage 60: The Paradox of the Global Warming Debate Passage Questions. ‘Answers and Explanations. 405 405 408 410 410 413, READING HomEworK PASSAGES Line 20 2 40 45 ‘The passage below is adapted from a 1911 novel that tells the story of a man whose dreams have been defeated by an unexpected turn of events. Thad the story bit by bit, from various people, and, as generally happens in such cases, each time it was a different story. If you know Starkfield, Massachusetts, you know the post-office, and yeu must have seen Ethan Frome drive up to it, drop the teins on his hollow-backed bay and drag himself across the brick pavement, and you must have asked who he was. Tt was there that, several years ago, I saw him for the first time; and the sight pulled me up sharp. Even then hhe was the most striking figure in Starkfield, though he ‘was but the ruin ofa man, It was not so much his great height that marked him, for the “natives” were easily singled out by ther lank longitude from the stockier foreign breed: it was the careless powerful look he had, in spite of a lameness checking each step like the jerk of a chain, There was something bleak and unapproachable in his face, and he was so stiffened and grizzled that I took him for an old man and was surprised to hear that he was not more than fifty-two. “He's looked that way ever since he had his smash- up; and that’s twenty-four years ago come next February.’ said Hermon Gow, who had driven the stage from Bettsbridge ‘o Starkfield in pre-trolley days and knew the chronicle of all the families on his lin. “The “smash-up” I gathered, had, besides drawing the red gash across Ethan Frome’ forehead, so shortened and warped his right side that it cost him a visible effort to take the few steps from his buggy to the post-office window. He used to drive in from his farm every day at about noon, and I often passed him in the porch or stood beside him while we waited on the motions of the distributing hand behind the grating, { noticed that, though he came so punctually, he seldom received anything but a copy of the Bettsbridge Eagle, which he put without a glance into his sagging pocket. At intervals, however, the post-master would hand him an envelope addressed to Mrs. Zeena-Frome, and usually bearing conspicuously in the upper left-hand corner the address of some manufacturer of patent medicine. Everyone in Starkfield knew him and gave him a greeting tempered to his own grave bearing; but his taciturnity was respected and it was only on rare occasions that one of the older men of the place detained him for a word. When this happened he would listen quietly, his blue eyes on the speaker's face, and 2 | orrmeaessn Pens ue answer in so low a tone that his words never reached me; then he would climb stiffly into his buggy, gather up the reins in his left hand and drive slowly away toward 50 his farm, “Tt was a pretty bad smash-up?” I questioned Harmon, looking after Frome’ retreating figure, and thinking how gallantly his lean brown head, with its shock of ight hair, must have sat on his strong, 55 shoulders before they were bent out of shape. “The worst kind? my informant assented. “More than enough to kill most men, But the Fromes are tough. Ethan'l likely touch @ hundred.” “Good God!" I exclaimed, A the moment Ethan 60 Frome, after climbing to his seat, had leaned over to assure himself ofthe security ofa wooden box, which he had placed in the back of the buggy, and I saw his face as it probably looked when he thought himself alone. “That man touch a hundred? He looks as if he 65 was dead and in hell now!” “Guess he’s been in Starkfield too many winters. ‘Most of the smart ones get away” said Harmon, “Why didn't he?” “Somebody had to stay and care for the folks. There 70 weren't ever anybody but Ethan, First his father—then his mother—then his wife” “And then the smash-up’ Harmon chuckled sardonically. “That's so. He had to stay then” 75 “Isee. And since then they've had to care for him?” Harmon thoughtfully passed his tobacco to the other cheek. “Oh, is always Ethan done the caring” ‘Though Harmon Gow developed the tale as far ashis mental and moral reach Fermitted there were 40 perceptible gaps between his facts, and I sensed that the deeper meaning of the story was in the gaps. But one phrase nevertheless stuck in my memory: “Guess he’s been in Starkfield too many winters.” Before my own time there was up T would come to 5 know what that meant. a “The passage is primarily concerned with ‘A) describing life in a rural community. B) comparing possible causes ofa tragedy. ©) examining an unusual figure. D) criticizing one man's habits. ‘As used in line 16, “checking” most nearly means A) studying, B) halting. ©) limping. D) verifying. ‘As revealed in the third paragraph (lines 11-20), the narrator's attitude toward Frome is best characterized as A) startled and intrigued. B) dismayed and disapproving. C) bewildered and fearful. D) saddened and protective, As used in line 54, “shock” most nearly means A) suprise, B) lack, ©) dash. D) mass. Harmon's statement inline 77 primarily serves to A) speculate on a probable outcome. B) contradict a presumption. ©). qualify a primary argument, D) justify a point of view. ‘Based on the passage, which of the following most accurately describes Ethan Frome? A) Rigid and reticent B) Rude and imposing ©). Decrepit and ignorant D) Generous and friendly Which choice provides the best answer to the previous question? A) Lines 4-12 (“Ifyou ... man”) B) Lines 33-50 (“I noticed ... farm") ©) Lines 59-67 (“Good god... Harmon”) D) Lines 69-77 ("Somebody . .. caring”) Ls peat Hae ‘The passage suggests which of the following about Harmon Gow? A) Heis originally from Bettsbridge. B) He does not stay in Starkfield for the winters. ©) He knows more about Ethan's story than he fees it is his place to say. D) He works at the Post Office in Starkfield. Bee eee ‘Which choice provides the best evidence for the answer to the previouis question? A) Lines 21-25 ("He looked .... line") B) Lines 51-58 (“It was... hundred”) ©) Lines 66-67 (“Guess ... Harmon”) D) Lines 78-83 (“Though Harmon... winters”) Which of the following can best be inferred from the narrator's statement “and I saw. . .in hell now” (lines 62-65)? A) The narrator believes that Fthan would be angry if he knew anyone was watching him. B) The narrator feels guilty for spying on Ethan. ©) The narrator presumes that Ethan wears a different face in public than in private 1D) ‘The narrator believes that Ethan is older than Harmon Gow thinks he is. erPmenvessent Hodge | 3 Answers and Explanations 1 C Throughout the passage, the narrator is describing his impressions of Ethan Frome, Find an answer that is consistent with this overview. Choice (A) is incorrect since itis too broad. Although the passage does imply a tragedy, there is never any comparison of such causes, so eliminate choice (B). Choice (C) fits, 0 ical. Choice (C) is the best answer. kkeep it. Eliminate chcice (D) since the passage is not 2, B Goto the second to last sentence of paragraph 3 and cross out the word checking and replace iewith your own word, A good replacement would be “slowing,” since his walking is described as /aneness and the phrase jerk of a chain implies he’s being held back by something. Eliminate choices (A) and (D) as they do not mean “slowing” in any way. Choice (C) almost works. While he may be limping, the word limping gives a description of his walk and does not work in the sentence in the same way as the word checking. Choice (B) is the best answer. 3. A__ Inthe third paragraph, the narrator refers to Frome as striking and that he was surprised that he wwas as young as he wis. Use POE to find the answer that best matches these descriptions. Choice (A) fits, so keep it for now. Eliminace choice (B) since hes not disapproving, Choice (C) is incorrect since the narrator is not fearful, nor is he saddened; eliminate choice (D) as well. Choice (A) is the best answer. 4 D Gow the second to last sentence of paragraph 7 and cross out the word shock and replace it ‘with your own word. A good replacement would be “large amount.” Choice (A) is the trap, since it marches the primary definition of shock, No evidence suggests Frome is losing hair, so eliminate choice (B). Choice (C) doesn't match “large amount,” so eliminate it. Choice (D) best fis the “large amount,” so itis the best answer. 5. B Inthe previous paragraph, the narrator asks if Frome’s family has to take care of him, but Har- ‘mon replies that its the other way around. Choice (B) matches this description. Eliminate choice (A) since no ‘outcome is discussed, ‘There isn’t much of an argument, so get rid of choice (C) as well. Choice (D) is incorrect because the Harmon is not trying to justify anything. Choice (B) is the best answer. 6 A__Use POR to find an answer choice that is most consistent with the information in the passage. Choice (A) fits since reticent means “not revealing one’s feelings” and rigid means “not willing to change.” Para ‘graph 6 discusses his tciturnity and habit of only picking up the Bettsbridge Eagle. Eliminate choices (B) and (D) because none of those descriptions fit Frome. While he is decrepit, he is the opposite of imposing, so eliminate choice (C). Choice (A) is the best answer 7. B___ Thesixth paragraph mentions that one of Frome’s characteristics is his taciturnity and also states that when at the store he rarely takes anything but the same newspaper, the Bettsbridge Eagle, These descriptions ‘match the correct answer from the previous question. Therefore, choice (B) is the best answer. Choices (A), (C), and (D) do no coordinate with any answers in the previous question and are, therefore, incorrect. 8. C_—_ Use POE wo find an answer choice that is most consistent with the information in the passage. “There is no exidence that Harmon was born in Bectsbridge, so eliminate (A). The only mention of winter is that Frome always stays during the winters, but no mention is made of Harmon, so choice (B) is incorrect. Keep choice (C) because the second to last paragraph states that Harmon's story has gaps and that the deeper meaning of the story was in the gaps, Ics stated that Harmon drove the stage coach from Bettsbridge to Starkfcld in the past, bbut there is no evidence to support that Harmon worked for the post office, so eliminate choice (D). Choice (C) is the best answer. 9. D___ The second to last paragraph justifies the previous answer because it suggests Harmon isnt tell ing everything about Ethan Frome. “The narrator suspects that the deeper meaning of the story was in the gaps, ‘Therefore, choice (D) isthe best answer. Choices (A), (B) & (C) do not coordinate with any answers in the previ- cous questions, and are therefore incorrect. 10. © Thenatrator states that he saw his face as it probably looked when he thought himself alone, o look for an answer that suggests that this isnt likely how Frome wants to be seen in public. Elimincte choice (A) since ‘we don't know if Frome is angry, as well as choice (B) since no mention is made of the narrator’ guile. Choice (© fis, so keep it for now. Choice (D) is incorrect since there is no evidence to conclude that Frome is older. Choice (C) is the best answer. ‘The following is an adaptation of Barbara Jordan's comments during the US. House Judiciary committee impeachment hearings in Washington, D.C. on July 25,1974. Itis a misreading of the Constitution for any member here to assert that for a member to vote for an article of impeachment means that that member must Line be convinced that the President should be removed 5 from office. The Constitution doestit say that. The powers relating to impeachment are an essential check in the hands of thebody of the legislature against and ‘upon the encroachments of the executive. The framers of the Constitutions were very astute to divide the two 10 branches of the legislature, the House and the Senate, assigning to the one the right to accuse and to the other the right to judge. They did not make the accusers and the judgers the same person, ‘We know the nature of impeachment. We've been 15 talking about it awhile now. Its chiefly designed for the President and his high ministers to somehow be called into account. Itis designed to “bridle” the executive if he engages in excesses. “It is designed as a method of national inquest into the conduct of public men.” The 20 framers confided in the Congress the power if need be, to remove the President in order to strike a delicate balance between a President swollen with power and ‘grown tyrannical, and preservation of the independence of the executive. 25 This morning, ina discussion of the evidence, we were told that the evidence which purports to support the allegations of misuse of the CIA by the President is thin. Were told that that evidence is insufficient. What that recital of the evidence this morning did not include 20 is what the President did know on June the 23% 1972. ‘The President did know that it was Republican money, that it was money from the Committee for the Re-Election ofthe President, which was found in the possession of one of the burglars arrested on June as the 17th, What the President did know on the 23% of June was the prior activities of E, Howard Hunt, ‘which included his participation in the break-in of Daniel Ellsberg’s psychiatrist, which included Howard Hunt’ participation in the Dita Beard ITT affair, which 40 included Howard unt’ fabrication of cables designed. to discredit the Kennedy Administration. ‘We were further cautioned today that pethaps these proceedings ought to be delayed because certainly there would be new evidence forthcoming from the 45 President of the United States. There has not even been. an obfuscated indication that this committee would receive any additional materials from the President. The committee subpoena is outstanding, and if the President ‘wants to supply that material, the committee sits here. 50 ‘The fact is that on yesterday, the American people waited with great anxiety for eight hours, not knowing ‘whether their President would obey an order of the Supreme Court of the United States. James Madison, from the Virginia ratification 55 convention. “Ifthe President be connected in any suspicious manner with any person and there be grounds to believe that he will skelter him, he may be impeached” ‘We have heard time and time again that the «0 evidence reflects the payment to defendant's money. ‘The President had knowledge that these funds were being paid and these were funds collected for the 1972 presidential campaign, We know that the President met with Mr. Henry Petersen 27 times to discuss matters «5 related to Watergate, and immediately thereafter met with the very persons who were implicated in the information Mr. Petersen was receiving, The words are: “If the President is connected in any suspicious manner with any person and there be grounds to believe that he 70 will shelter that person, he may be impeached. James Madison again at the Constitutional Convention: ‘A President is impeachable if he attempts to subvert the Constitution” The Constitution charges the President with the task of taking care that the 75 laws be faithfully executed, and yet the President has counseled his aides to commit perjury, willfully disregard the secrecy of grand jury proceedings, conceal surreptitious entry, attempt to compromise a federal judge, while publicly displaying his cooperation with «0 the processes of criminal justice. Ifthe impeachment provision in the Constitution of the United States will not reach the offenses charged here, then pethaps that 18th-certury Constitution should be abandoned to a 20th-century paper shredder! a5 Has the President committed offenses, and planned, and directed, and acquiesced in.a course of conduct which the Constitution will not tolerate? Thats the question. We know that. We know the question. We should now forthwith proceed to answer the question. 0 Itis reason, and not passion, which must guide our deliberations, guide our debate, and guide our deci RES “The position that Jordan takes is primarily that of A) an academic authority providing a critical perspective. B) a jurist providing a specific legal interpretation. ©) acritic discounting the position of the uninformed public. D) allegal advocate defending the President’ actions. es Ei In the passage, Jordan draws a distinction between ‘A) how tyrants and presidents take power of a nation. B) grounds for impeachment and removal from office. ©) the actions of the President and his high ministers. D) the nature of administration and smaladministration, oS eS Which choice provides the best evidence for the answer to the previous question? A) Lines 1-5 (“Itis... office”) B) Lines 15-17 (“Itis... account”) ©) Lines 19-24 ("The framers... executive”) D) Lines 31-35 (“The President ... the 17th") ey Gite ‘The passage suggests that a President would be impeachable for each of the following EXCEPT A) Having a relationship witha criminal such that the President might shield him from prosecution B) Engaging in the bribery of a judge on a federal appeals court ©) Consistently asscciating with a person who had previously been convicted of several felonies D) Advising a member ofhis Cabinet to knowingly lie before a Senate committee a ace ‘The authors atitude toward those who cautioned the committee can be best described as A) skeptical, B) suspicious. ©) amenable. D) annoyed. ‘As used in line 17, “bridle” most nearly means A) investigate. B) check ©) judge. D) swell, eR Jordan mentions =e Peterson (line 64) primarily in order to ‘A) explain the importance of the citation of James Madison that follows. B) name a witness who will likely appear in the future impeachment hearings. ©) concede that one particular cherge against the President is unsubstantiated. D) describe an interaction which might serve as evidence supporting impeachment. aS Which choice provides the best ev answer to the previous question? A) Lines 42-45 (“We were... States’) B) Lines 50-53 (“The fact... States’) ©) Lines 54-58 (“James ... impeached”) D) Lines 71-73 (“James ... Constitution”) ovmemewonirhotn uc | 7 ‘As used in line 82, “reach” most nearly means |The principal shetorical effect of the phrase “If A) cover the impeachment .. . paper shredder!” can be best described as B) arriveat, |A) asincere suggestion about what the Congress ought ©) grasp. to do with the 18 century legal document Jordan D) move toward. has mentioned. B) abyperbolic claim meant to emphasize the degree to which the evidence against the President fits the impeachment criteria. ©) show that Jordan believes the impeachment proceedings against the President to be entirely ridiculous. D) caution the committee that they ought to continue to wait for more evidence from the President and his couneil. THIS PAGE IS LEFT INTENTIONALLY BLANK. orrmeheaton Pilon ue | 9 0 Answers and Explanations 11. B__Jordaa devotes a good deal of attention to analyzing the legal concepts at play in this passage, including the legal definition of impeachment in lines 14-24. She describes the legal features of impeachment without criticizing or disputing them, so she is not providing a critical perspective. Therefore, (A) is not sup- ported by the passage. As Jordan's focus is on matters of legal definitions and law-breaking, she is providing an interpretation of law, which supports (B). While Jordan does suggest that che President is presenting a filse image to the public in lines 73-80, itis too extreme to say that the public is uninformed or disagrees with Jordan. Thus, (©) is not correct. Jordan suggests her suspicions of the President's involvement in illicit actions in lines 59-70, indicating that she does not support his actions. Therefore, (D) is incorrect, 12, B___ Jordan does draw a comparison between the actions ofa tyrant and an overly powerful President in lines 19-24, but shedoes not discuss how tyrants take power of a nation. Therefore, (A) is not supported by the passage. Jordan explains the grounds for impeachment in lines 54-58, suggesting that there need be only evidence of the Presi 35-41. Jordan clarifies later chat this evidence is sufficient for an impeachment while the question of removal it’s connection to suspicious matters in order to impeach, and she provides that evidence in lines from office remains ta be seen; that distinction supports (B). Jordan's only mention of the high ministers is in lines 15-17, when she groups the high ministers together with the President and discusses their actions together. “Therefore, (C) is incomect. Jordan does not mention the concept of the nature of administration in this passage, 80 (D) is incorrect. 13, A The answer to the previous question states that Jordan draws a distinction between grounds for impeachment and removal from office. Choice (A) discusses this distinction, so choice (A) is the correct answer. ‘Choices (B) and (C) discuss the nature of impeachment, but not the difference between impeachment and re~ moval from office, so these choices are not correct. Choice (D) discusses the specific allegations against President Nixon, so choice (D) is incorrect. 14. © _ Jordan indicates that a President is impeachable if he may shelter a suspicious person in lines 54-58, which demonstrates that (A) is included in the passage and is not the correct answer. Jordan describes compromising a federal judge as an impeachable offense in line 78, and bribery is an act of compromising an official, which demonstrates that (B) is included in the passage and is not the correct answer. While Jordan in- dlicates that the President may be impeachable if he shelters a criminal, she does not indicate that the President may not be associated with a criminal at all; chis means that (C) is not included in the passage and is the correct answer. Among other impeachable actions, Jordan also references the President counseling his staff members 0 lie in lines 75-77, which demonstrates that (D) is included in the passage and is not the correct answer. 15. A Jordan describes chose who cautioned the committee in lines 42-53. She indicates that their prediction, tkat the President would provide new evidence, is doubsful because the President has been given the ‘opportunity to supply additional material and has not done so. She does not believe that the President will pro- vide additional evidence, and her atcieude coward those who cautioned the committee is disbelieving. Skeptical ‘means the same as dsbelieving, so (A) is correct. Choice (B) suggests that Jordan suspects il intentions o foul play on the part of those who cautioned the committee, which is too extremes (B) is incorrect, Choice (C) suggests that Jordan is friendly and accommodating toward those who cautioned the committee, which is not correct; (C) isnot correct. Choice (D) icates that Jordan is ivitated with those who cautioned the committce or is angry toward them, which is too excreme; (D) is not supported by the passage. 16, B___In the sentence in question, Jordan continues her discussion of the nacure of impeachment and says that itis designed ro bridle the executive if he engages in excesses. The use of the word excese indicates that it is possible forthe executive to go too far, suggesting that there are specific boundaries ofthe executive's actions. This means that the executive must be kept within the boundaries of his acceptable actions, and ke must be stopped from excceding the boundaries. This means that bridle means stop, and che correct answer must match stop. Since investigate, judge, and swell do not match stop, you can eliminate (A), (C), and (D). Since check means step, (B) is the correct answer. 17. D___Jordan utilizes the citation of James Madison to explain che legal implications of the reference to Henry Peterson and its importance to the President's impeachment, as in lines 54-58. She does not reference Henry Peterson to make the citation of James Madison more important. Therefore, (A) is not correct, Jordan does not reference witnesses who may appear in impeachment hearings, so (B) is incorrect. Jordan references Henry Pererson among other claims against the President's innocence, as in lines 59-63. Therefore, she groups the ac- cusation of misconduct with Peterson among other offenses she believes, so she does nor think the accusation is unsubstantiated. Thus, (C) is incorrect. Jordan does describe the interaction with Henry Peterson among other claims of misconduct, and she immediately follows the statement with the reference to James Madison indicating that such claims are grounds for impeachment. ‘Therefore, (D) is correct. 18. € Choice (A) discusses the possibility of formal impeachment trials, which were not discussed str- rounding Henry Peterson; thus, (A) is not correct. Choice (B) expresses a claim against the President's conduct, that is critiqued in the same way that Jordan «1 riques the President's conduct with Henry Peterson. However, Jordan does not reference Henry Peterson to prove that the President did not deliver new evidence; therefore, (B) is incorrect. Jordan explains thatthe President associated with Henry Peterson in a suspicious manner, and (C) is used by Jordan co explain that these actions are grounds for impeachment. Therefore, (C) is correct. Choice (D) ‘explains a reason for the President to be impeachable, but that reason is not connected to the claims regarding Henry Peterson; thus, (D) is not correct. orPaEsextoniP lin | 11 2 19. A__ Inthe sentence in question, Jordan indicates that the impeachment provision in the Constitu- tion should be shredded if it does not reach the offenses charged. This means she believes that the impeachment provision should reach the offenses charged. Inthe paragraph before the sentence in question, ines 71-80, Jordan, describes various actions ofthe President that she considers impeachable. Jordan indicates that the impeachment provision should reach these charges and consider them impeachable. This suggests that the impeachment provi- sion must address the charges as impeachable actions. A good word co replace reach would be address and the cor- rect answer must matca addres. Since arrive at, grasp, and move toward do not match addres, you can eliminate (B), (©, and (D). Since cover means address, (B) is the correct answer. 20. B__In the paragraph before the sentence in question, lines 71-80, Jordan indicates that the Presi- dent's actions are impcachable, which suggests that she believes they will be included in the Constitution's im- peachment provision, Therefore, she is not sincerely suggesting that the impeachment provision is useless and that the Constitution be destroyed, so (A) is incorrect. Jordan considers the President’ actions worthy of im- peachment and therefere believes the impeachment provision to be true; she would not sincerely want to destroy ic, so her suggestion to do so indicates her certainty chat the impeachment provision includes the actions she has detailed, so that she will not have to make good on her threat to destroy it. Therefore, (B) is supported by the passage. Jordan has indicated with certainty that the case o impeach the President is strong, and she stands as one arguing in favor of impeachment. Therefore, (C) is not correct, Jordan describes the fact that the committee has been cautioned in lines 42-45, but she indicates that the committee was cautioned by another pa:ty, not by herself. Thus, (D) is not correct. THIS PAGE IS LEFT INTENTIONALLY BLANK. CerPnchvatn steps | 3 “ ‘This passage isan excerpt from incognito: The Secret Lives ofthe Brain by David Eaglemen in 2012 by Vintage. ‘The brain isa complex system, but that doesn’t mean its incomprehensible. Our neural circuits were carved by natural selection to solve problems that our ancestors tine faced during our species’ evolutionary history. Your 5 brain has been melded by evolutionary pressures just as your spleen and eyes have been. And so has your consciousness. Consciousness developed because it ‘was advantageous, but advantageous only in limited amounts. 10 Consider the activity that characterizes a nation at any moment. Factories churn, telecommunication lines buzz with activity, businesses ship products. People eat constantly. Sewer ines direct waste. Al across the great stretches of land, police chase criminals. Handshakes 15 secure deals. Lovers rendezvous, Secretaries field calls, teachers profess, athletes compete, doctors operate, bus drivers navigate. You may wish to know whats happening at any moment in your great nation, but you can‘t possibly take in all the information at once. 20 Nor would it be useful, even if you could, You want a summary. So youpick up a newspaper—not a dense paper like the New York Times but lighter fare such as USA Today. You wont be surprised that none of the details of the activity are listed in the paper; after all, 25 you want to know the bottom line. You want to know that Congress just signed a new tax law that affects your family, but the detailed origin of the idea— involving, lawyers and corporations and filibusters— isntt especially important to that new bottom line. And you 30 certainly wouldn't want to know all the details of the food supply of the nation—-how the cows are eating and how many are being eaten—you only want to be alerted if there’ a spike of mad cow disease. You dor't care how the garbage is produced and packed away; you only care 45 ifit’s going to end up in your backyard. You dont care about the wiring and infrastructure of the factories; you only care if the workers are going on strike. That’s what ‘you get from reading the newspaper. Your conscious mind is that newspaper. Your brain 40 buzzes with activity around the clock, and, just like the nation, almos: everything transpires locally: small groups are constantly making decisions and sending out ‘messages to other groups. Out of these local interactions ‘emerge larger coalitions. By the time you read a mental 45 headline, the important action has already transpired, the deals are done. You have surprisingly little access to what happenec behind the scenes, Entire political ‘movements gain ground-up support and become ‘unstoppable before you ever catch wind of them as a 50 feeling or an intuition or a thought that strikes you. You'ee the lst one to hear the information. However, you're an odd kin¢ of newspaper reader, reading the headline and taking credit for the idea as though you thought of it first. You gleefully say, “I 55 just thought of something”, when in fact your brain performed an enormous amourt of work before your ‘moment of genius struck. When an idea is served up from behind the scenes, your neural circuitry has been ‘working on it for hours or days or years, consolidating 60 information and trying out new combinations, But you take credit without further wonderment at the vast, hidden machinery behind the scenes. ‘And who can blame you for thinking you deserve the credit? The brain works its machinations in secret, 65 conjuring ideas like tremendous magic. It does not allow its colossal operating system to be probed by conscious cognition. The brain runs its show incognito. 12] ‘As used in line 5, “molded” mest nearly means A) curved. B) decayed. ©) evolved. D) pressed. Based on the passage, itcan be reasonably inferred that the relationship between the brain and conscious thinking ishest characterized as which of the following? A) A cashier at the counter and a cook in the kitchen B) A worldly newspaper journalist and an aspiring novelist ©) A battle of wits between two world-class chess champions D) Aself-involved auteur anda film crew that does all the work ‘The primary purpose of the second paragraph isto show that ‘A)_ your neurological framework is alert to all dangers. B) you try to summarize every detail listed in the newspaper. ©) your neural circuitry is processing ideas at all times. ) your conscious mind gets limited access to sensory information for a reason. “The principal rhetorical purpose ofthe phrases in lines 25-37 (“You want to know that Congress... . you only care if the workers are going on strike.”) is to A) support the argument in the last paragraph. B) resolve a contradiction in neurological versus sociological theory. C) offer examples of how society needs different elements to work D) provide metaphors that display specific and overabundant desails. ae Which choice provides the best evidence for the answer to the previous question? A) Line 20 ("Nor would ... could”) B) Lines 37-38 (“That's what .. . newspaper") C) Line 39 ("Your. .. newspaper”) D) Lines 57-60 (“When an ‘combinations”) Bee As used in line 46, “surprisingly” most nearly means A) unexpectedly. B) frighteningly. C) surreptitiously. D) passably. ‘The author mentions “odd ... newspaper reader” (ine 52) primarily in order to A) criticize your limited comprehension skills. B) support the thesis that the brain is a newspaper, ©) introduce the final aspect of the metaphor. D) contradict a prior assumption. LE According to the passage, a nation is characterized by each of the following EXCEPT A) professors egressing. B) police pursuing ©) factories creating. D) Congress legislating. ‘The passage suggest that the brain A) is carved up into separate parts, B) functions on a level that's separate from the conscious mind. ) prevents the conscious mind from reaching its full potential, D) hides feelings that are upsetting and unsettling. ‘Which choice provides the best evidence for the answer to the previous question? A) Lines 22-25 (“Times but ... line”) B) Lines 43-44 (“Out of... coalitions”) ©) Lines 54-57 (“You gleefully... struck’) D) Lines 64-65 (“The brain ... magic”) ‘The primary purpose of the passage as a whole is to ‘A)_ hypothesize that our consciousness is limited. B) characterize our neurological operations as a dual system. (©) suggest that the workings of the brain are a secret to our consciousness. D) propose that we give the brain the credit it deserves. nue | Answers and Explanations 32, C The word molded is used to show that the brain developed according to environmental needs, thus (C) is correct. The brain is curved, (A), but molded refers to how icwas created, not how itended up looking. Choice (B) is wrong since there was no rotting involved and (D) is a trap due to the word ‘pressures’ ater in the sentence, 33. D_ _ The relationship between the brain and the conscious mind is one of a complex system that manages many details and ideas—the brain—and of a conscious mind that’s oblivious to the work involved, ryt takes the credit, so (D) is correct, since the film director—auteur—doesn't realize that all the work has been done by the crew. The cashier isn't necessarily unsure of what the cook is doing, so (A) is incorrect. They are not differenc in age or expsrience, so (B) is incorrect. As previously described, the brain and conscious mind are not presented as equals, wiich makes (C) incorrect 34. D-—_—_Chice (A) is too extreme, since you aren't alert to all dangers, so it’s incorrect, “The passage states that you dont try to process every detail, making (B) incorrect. Choice (C) is true, but the discussion about neural circuitry processing ideas is in paragraph four. Choice (D) is the main purpose of the second peragraph— to show why the conscious mind only processes limited information. 35. D___ Choice (A) addresses the main idea of the passage as articulated in the last paragraph, but doesn’t address the way in which the author makes this point in the lines in question, so eliminate (A). Choice (B) is incorrect because it ions resolving a contradiction that is not mentioned in the paragraph, Choice (C) takes the phrases as literal examples of what society needs to function, when these phrases actually have a metaphori- cal purpose. Choice (D) addresses the rhetorical choice that the author made by using repetitive clauses to show repeatedly how much extra information there is compared to how much your conscious mind needs. Thus, (D) is correct. 36. A__ The passage states that consciousness is only advantageous in limited amounts and then it says in paragraph two that You may wich to know whats happening at any moment in your great nation, but yew cant pos- sibly sake in all the information at once, This is what makes (A) correct since it addresses the fact that it wouldn't be useful for the conscious mind co have all of the information. Choices (C) and (D) ate in the passage, but the purpose of the second paragraph isnt to say the conscious mind is the newspaper or to explain how the brain works behind the scenes; the purpose of the second paragraph is to show why the conscious mind needs to sore through information and only take in limited amounts of it. Choice (B) is vague, so, while it hints atthe sores of details the conscious mind wants, it doesn‘ explain the purpose. 37. A "Theauthor explains that the conscious mind doesn't get the information from the brain until the brain has done a lor of work on its own. This is contrary to how most people see the brain working. We assume tha we ate aware of how our brains work. Thus, it’s unexpected that we don’t know about the inner working of the brain until they become conscious ideas—making (A) correct. Choice (B) is extreme, since it’s not necessar- ily scary. Cheive (C) means secretive, which can refer to the brain, but doesnt refer co how most people think about the thought process. Choice (D) means sufficiently and that has nothing to do with being unexpected and is incorrect. 38. Cc ‘Choice (A) is wrong because the author doesn't criticize the brain's ability to understand what it takes in, while (B) is wrong since the brain itself isn che newspaper in the metaphor. The brain actually gathers all the information that consciousness doesn't wane filling up the metaphorical newspaper. Choice (D) is incorrect since the author doesn't contradict any assumptions made in the prior paragraphs. ‘Thus, (C) s correct, 39. A Choices (B), (C), and (D) are mentioned in lines 10-16 as police chase criminals, Factories churm, and Congress just signed a new tax laws Choice (A) seems to refer to teachers profesin the aforerrentioned lines, but cegressing means exiting, which is not the same as professing. 40. B_ Choice (A) uses carved in a way that sounds as if the brain is actually cut up, so is incorrect Choice (C) implies thatthe brain somehow works against the conscious mind, bue the brain helps our perception by only leting some details through. The passage also doesn't say that the brain hides feelings that are upsetting, 0 (D) isinconect. Choice (B) is correct, since the passage says that the brain hides its working from the conscious mind, 41. D___Choice (D) discusses how the brain operates in secret, so (A), (B), and (C) are incorrect. Choice (A) hints at how the brain leaves information out, but doesn't address the fact that che conscious mind is unaware of what's missing. Choice (B) refers to how the brain works, but on a too-narrow scale. Choice (C) hints that the conscious mind is unaware, but it seems more like a matter of timing, while (D) clarifies that the brain operates in secret and is therefore correct. 42, © “Lhe main idea is that the brain works in secret, helpful ways, while our conscious minds are un- ave of all tat we've gathered. ‘Thus, (C) is correct. Choice (A) is wrong since the limits of cur conscious mind are only a pate of the passage’s purpose. Choice (B) recycles the phrase neurological operations from the passage, bbut doesnt address the main idea. While the passage describes the vast amount of work the biain does, the point is not to convince us to give it more credit. So (D) is incorrect, ovmthenenPtie us| 17 This passage is adapted from a State of the Union Address delivered by US. President Richard Nixon on January 22, 1970. In the passage, Presicent Nixon discusses steps that he plans +o take to balance the country’s budget and to reduce crime. Only with the cooperation of the Congress can ‘we meet this highest priority objective of responsible government. We are on the right track. Lune We had a balanced budget in 1969. This 5 administration cut more than $7 billion out of spending plans in order to produce a surplus in 1970, and in spite of the fact that Congress reduced revenues by $3 billion, I shall recommend a balanced budget for 1971 But I can assure you that not only to present, but to 10 stay within, a balanced budget requires some very hard decisions. It means rejecting spending programs which ‘would benefit some of the people when their net effect ‘would result in price increases forall the people. Itis time to quit putting good money into bad 15 programs. Otherwise, we will end up with bad money and bad programs, I recognize the political popularity of spending programs, and particularly in an election year. But unless we stop the rise in prices, the cost of living for 20 millions of American families will become unbearable and government’ ability to plan programs for progress for the future will become impossible. In referring to budget cuts, there is one area where I have ordered an increase rather than a cut—and that is 25 the requests of those agencies with the responsibilities for law enforcement. ‘We have heard a great deal of overblown rhetoric during the sixties in which the word “wat” has perhaps too often been used—the war on poverty, the war on 30 misery, the war on disease, the war on hunger. But if there is one area where the word “wat” is appropriate it isin the fight against crime, We must declare and ‘win the war against the criminal elements which increasingly threaten our cities, our homes, and our 35 lives. ‘We have a tragic example of this problem in the nation’s capital, for whose safety the Congress and the Executive have the primary responsibility. I doubt if many Members of this Congress who live more than a 40 few blocks from here would dare leave their cars in the Capitol garage and walk home alone tonight. Last year this edministration sent to the Congress 13 separate pieces of legislation dealing with organized crime, street crime, narcotics, and crime in the District 45 of Columbia None of these bills has reached my desk for signature. Tam confident that the Congress will act now to adopt the legislation I placed before you last year. We s0 in the Executive have done everything we can under existing law, but new and stronger weapons are needed in that fight. It is true that state and local law enforcement agencies are the cutting edge in the effort to eliminate 55 street crime, burglaries, and murder. However, my proposals to you have embodied my belief that the federal government should play a greater role in ‘working in partnership with these agencies. ‘That is why 1971 Federal spending for local law «69 enforcement will double that budgeted for 1970. ‘The primary responsibility for crimes that affect, individuals is with local and state rather than with federal government. Butin the field of organized crime and narcotics, the federal government has a ‘65 special responsibility it should fulfill. And we should ‘make Washington, D.C,, where we have the primary esponsibility, an example to the ation and the world of respect for law rather than lawlessness. Us. Federal Budget Surphu/Dafcl (FY) 1930-2000 150 . 100 f “eso te40—160~—1060~—«w7O~—«n080 «000 «2000 year ‘This graphic from the State University of New York depicts the US. Federal budget between 1930 and 2000, Any number above 0 on the y-axis indicates a surplus, while any number below itingicates a defi. 121] As used in line 12, “net” most nearly means A) budgeted B) meshed, ©) final D) biased, ‘The position Nixon takes is primarily that of A) ahead of state explaining governmental failures. B) aleader recommending various courses of action. ©) anoptimist predicting the country’ future successes. D) adisinterested onlooker presenting the facts of a situation. ‘The passage most strongly suggests that the proposed budget will ‘A) cause prices across the nation to rise. B) result in bad programs paid for by counterfeit funds. ©) increase Nixon’ political popularity. D) eliminate some programs that are helpful to US. citizens. Which choice provides the best evidence for the answer to the previous question? A) Lines 9-13 (“But I. .. people”) B) Lines 15-16 ("Otherwise ... programs”) ©) Lines 17-18 ("I recognize ... year”) D) Lines 18-22 (*Butunless.. . impossible”) In lines 27-30, what is the most likely reason that Nixon mentions the “overblown rhetoric” of the sixties? A) To respond to claims that the country should not invest in the fight against starvation, sickness, and destitution B) To argue that budget cuts should not affect agencies that provide for individuals suffering from poverty and sickness ©) To draw a distinction between the way in which the ‘word “war” has been used in the past and the way in which he uses the word “war” D) To acknowledge that the while crime is increasingly common in cities, the nation’s war against debt and. disease has been successful ‘The primary thetorical effect of the phrases “our cities, our homes, and our lives” (line 34) isto A) analyze the ways in which towns, residences, and families are being torn apart by war. B) indicate that Nixon believes tha: US. policies affect average Americans in three separate ways. ©) suggest three areas that congress should examine to determine the common sources of criminal tendencies. D) express with increasing emphasis the extent to which crime affects US. citizens. ‘What is the most likely reason that Nixon mentions the fact that Members of Congress would not “dare leave their cars in the Capitol garage” (lines 40-41)? A) To emphasize the need for better lighting and security systems in local parking facilities B) Toillustrate the importance of passing legislation that would help to reduce crime ) To support the claim that many Members of Congress fail to take responsiblity for providing for the safety of the nation D) To demonstrate that the nation’ capital is becoming increasingly less safe than many of the other cities in the nation Smet ssegs UC | 18 wt 7 Which choice provides the best evidence for the answer to the previous question? |A) Lines 27-30 ("We have B) Lines 32-35 ("We must... lives") ©) Lines 53-55 ("It is... murder”) D) Lines 61-63 (“The primary ... Government”) hunger”) ‘As used in line 48, “adopt” most nearly means A) foster B) confide. ©) approve. D) seize. Based on the passage, which of the following describes the relationship between the state and federal government in fighting crime? 'A) The state competes with the federal government in the fight against violent crime and narcotics. B) ‘The state is one of the organizations chiefly responsible for fighting crime, but the federal ‘government has the duty to fight specific types of ©) The federal government is responsible for fighting crimes that the state fails to successfully combat. D) The federal government prosecutes individuals who commit crimes against the nation, while the state prosecutes individuals who commit local crimes. 20 | DrPReaseton gs Le Using information from the passage and the graph, it can reasonably inferred that A) by cutting more than $7 billion in spending, the Nixon administration was able to produce a surplus for the first time since 1930. B) despite the plans he outlined in this speech, Nixon was unable to balance the US, budget in 1971. ©) Nixonis spending strategy ensured that the US. never again had a budget deficit. 1D) increasing the budget for local law enforcement was the sole cause for the many deficit years following 1971 THIS PAGE IS LEFT INTENTIONALLY BLANK. ormneheaion® dens UC | 21 Answers and Explanations 21. © The sentence in which the word net appears reads, it means rejecting spending programs which would benefit some of the people when their net effect would result in price increases for all the people. Based on the context, a good word to put in place of the word met might be end or total. Since budgeted, meshed, and biased al have meanings that are different from the word end or total, you can eliminate (A), (B), and (D). Only the word in (C), final, has a meaning similar to end or total, so (C) is the correct answer. 22. B While Nixon could certainly be considered a governmental head, he is not mainly explaining governmental failures. Instead he discusses problems facing the government, but recommends courses of action to fix chose problems, Thus, (A) is incorrect. Since Nixon is a leader, and in his speech he discusses problems ‘within the country ard recommends steps that the government should take, (B) accurately describes his stand and is the correct answer. Nixon does not predict the country’s successes in any way, so (C) is incorrect. Finally, Nixon is the president of the country and therefore is not a disinterested onlooker. ‘Therefore, you can eliminate (D). 23. D__ Inthe fifth paragraph, Nixon discusses how important it is to stop the rie in prices. Therefore, it is unlikely that he would propose a budget that would cause prices across the nation to rise. Thus, you can elimi- nate (A). In the fourth paragraph, Nixon does mention bad money and bad programs. However, he is not literally discussing counterfeit funds, nor is there any evidence that the proposed budget will create bad programs. Thus, (B)is incorrect. In the fifth paragraph, Nixon does mention the political popularity of spending program. However, he does not state that the budget in question will increase his own popularity, so (C) is not the correct answer. “The third paragraph states that to stay within a balanced budget requires some very hard decisions It means rejecting spending programs which would benefit some ofthe peopl. Thus, the passage does suggest that the propesed budget will eliminate some programs that are helpful to U.S. citizens, and (D) is the correct answer 24. A Theanswer to the previous question was (D): the proposed budget will eliminate some pro- grams that are helpful to U.S. citizens. The lines referenced in (A) indicate that a balanced budget means rejecting spending programs which would benefit some of the people when their net effect would result in price increases for all she people. This information in duese lines provides support for the answer to the previous question. Thus, (A) is the correct answet. The information in (B) does not mention the elimination of programs that are helpful to USS. citizens, so (B) is incorrect. The information in (C) mentions spending programs, but does nor discuss the fact that the proposed budget might eliminate some programs, so (C) is incorrect. The information in (D) does reference programs for progress, bc it suggests that unless the government can stop rises in prices, the government's ability to plan for these types of programs will become impossible. Thus, this does not suggest that the budget will eliminate some programs that are helpful to U.S. citizens, and is therefore incorrect, 22 | errneencon tease 25. C__ The passage does not mention any claims that the country should not invest in the fight against starvation, sickness, and desticution, so (A) is incorrect. Nixon states in the sixth paragraph that there i one area where Ehave ordered an increase rather than a cut ~ and that is the requests of those agencies with the responsibility of law enforcement. Since this paragraph indicates that in areas other than law enforcement, Nixon has ordered budget cuts, there is no evidence thac he is arguing that budget cuts should not affect agencies that provide for individuals suffering from poverty and sickness. Thus, (B) is incorrect. In referring to the overblown rhetoric dur- ing the sixties, Nixon mentions that the word “war” has perhaps too often been used. He then give some examples of the way in which the word war has been used too frequently, but then states that ifchere is one area where the ‘word “war” is appropriate itis in the fight against crime. Hie then goes on to discuss reasons tha: the country must fight a war against crime. Therefore, Nixon must have introduced the overblown rhetoric of the sixties in order to show a difference between the way the word war has been used in the past, and the way in which he uses was, and (O) is the contect answer. Nixon never states that che war against debe or disease has been successful, so you can dliminate (D) 26. DI the relevant part of the text, Nixon is discussing the ways in which erime, not war, has af- fected Americans. Thus, you can eliminate (A). Similarly, (B) focuses on the effects of U.S. policies rather than con the effects of crime, so (B) is incorrect. Nixon does not say that cities, homes, and lives cause crime, only that they are affected by crime, so (C) is not the correct answer. Since Nixon lists multiple ways in which crime affects Americans, the phrase in question serves to emphasize the extent to which crime effects them, and (D) is the cor- rect answer. 27. B___ Shortly before Nixon mentions the fact that Members of Congress would not dave eave their cars in the Capitol garage and walk home at night, he staes that the nation must declare and win the war against the criminal elements which threaten our cities, our homes, and our lives. He then states that the problems in the nation’s capital provide a eragic example ofthis problem. Shortly after he mentions the fact that Members of Con- gress would be unlikely to leave their cars in the Capitol garage, he points our that 13 separate pieces of crime- telated legislation had been submitted in the past year, but that none of those pieces of legilation reached his desk. He follows this up by declaring, Zam confident that the Congress will act now to adopt the legislation I placed before you last year. Therefore, considering what he says immediately before and immediately after the statement in question, he must mention the Members of Congress and their reluctance to leave their cas in the Capicol ‘garage in order to point out the danger which crime poses to the country, and to show the importance of passing (A) might help to deter crime in parking garages in particular, Nixon’ argument is about crime in gencral throughout the nation, so (A) is incortect. Choice (B) fits what you're looking for, and isthe correct answer. Choice (C) does relate tothe safety of the nation, but since Nixon is not at- new anti-crime legislation, While the measures tempting to place blame on the Members of Congress and is instead simply urging them to take action, (C) is not the correct answer, Nixon does not compare the capital with other cities in the nation, so you can eliminate (D). a 28. B_ —_The answer to the previous question was (B), 10 illustrate she importance of passing legislation that would help to reduce erime. Therefore, you need an answer quote from the passage which makes a similar state- ‘ment. The thoughts expressed in (A) do not relate to crime, s0 you can eliminate (A). The quote in iB) is very similar to the answer to the previous question, so (B) is the correct answer. Choices (C) and (D) both relate to concessions that Nixon makes regarding the need for che government to pass anti-crime legislation; he admits that law enforcement zgencies are currently cucting edge, and that the primary responsibility to fight erime rests with local and State agencies. However, his primary point is the point argued in the answer to the previous ques- tion: Congress should pass legislation to help reduce crime. Since neither (C) nor (D) makes this point, both choices are incorrect. 29. © The sentence in question says, “I am confident that the Congress will act now to adopt the leg- islation I placed before you last year” Based on the context, a good word to put in place of adope might be accept cor pass. The words foster, confide, and seize all have meanings different from accept ot pass you can eliminate (A), (B), and (D). Since approve is similar in meaning to accept or pass, the correct answer is (C) 30. B__ The final paragraph of the passage does state that the federal government has a responsibility fight organized crime and narcotics, but it does not indicate that the State and the federal government compete in this fight. Therefore, (A) is incorrect. The final paragraph does say, however, that the primary responsibility for crimes that affect individuals is with the local and state rather than the federal governmens, supporting, the statement that the state is chiefly responsible for fighting crime. ‘The final paragraph also states that in the field of organized crime and narcotics, the federal government has a special responsibility. Therefore, the federal government has the duty to fight specific types of crime—organized crime and narcotics. Choice (B) is supported by the passage, and is the correct answer. The passage does not provide any evidence for the idea chat che federal government must fight crimes that the state fails to combat, so you can eliminate (C). "The passage also does not suppert the idea that the federal government pursues those who commit crimes against the nation, while the state pursues those who commit local crimes, so (D) is incorrect. 31. B__ Inthe passage Nixon recommends « balanced budget for 1971. However, the graphicshows that the USS. budget had a deficit in 1971, which implies that Nixon didn’t meet this goal. Choice (B) best summa- rizes this contrast. Although the passage does state that the Nixon cut $7 billion in spending, the graph shows several surplus years since 1930, Therefore, (A) is incorrect. The graph shows many years of defici: following Nixon's announcemert of his spending strategy, so (C) can be eliminared. And while the U.S. budget cid suffer in the years following this speech and the speech did mention raising the law enforcement budget, itis unreasonable to assume that increasing the budget for local law enforcement was the sole cause, Therefore, (D) is incorrect. ‘oTPREduesni gs THIS PAGE IS LEFT INTENTIONALLY BLANK. on etcaen aigs Ut | 25 The passage below is adapted from the autobiography of Joseph Jefferson, an influential late nineteenth century actor In the essay below, .fferson discusses the importance of balancing spontaneity and preparation in the art of acting, Thave seen impulsive actors who were so confident of their power thet they left all to chance. This is 1a dangerous course, especially when acting a new tine character. I will admit that there are many instances where great effects have been produced that were entirely spontaneous, and were as much a surprise to the actors who made them as they were to the audience ‘who witnessed them; but just as individuals who have exuberant spirits are at times dreadfully depressed, s0 10 when an impulsive actor fails to receive his inspiration he is dull indeed, and is the more disappointing because of his former brilliant achievements In the stage management of a play, or in the acting ofa part, nothing should be left to chance, and for the 15 reason that spontaneity, inspiration, or whatever the strange and delightful quality may be called, is not to be commanded, or we should give it some other name. Itis, therefore, better that a clear and unmistakable outline ofa character should be drawn before an actor 20 undertakes a new part. Ifhe has a well-ordered and an artistic mind isis likely that he will give at least a symmetrical and effective performance; but should he ‘make no definite plan, and depend upon our ghostly friends Spontancity and Inspiration to pay him a visit, 25 and should they decline to call, the actor will be in a maze and his audience ina muddle Besides, why not prepare to receive our mysterious friends whether they come or not? If they fail on such an invitation, we can at least entertain our other guests, 30 without them, and if they do appear, our preconceived plans will give them a better welcome and put them more at ease ‘Acting under these purely artificial conditions will necessarily be cold, but the care with which the 35 partis given will atleast render it inoffensive; they are, therefore, primary considerations, and not to be despised. The extibition, however, of artistic care does not alone constitute great acting. The inspired warmth of passion in tragedy and the sudden glow of humor 40 in comedy cover the artificial framework with an impenetrable veil this is the very climax of great art, for which there seems to be no other name but genius, Itis then, and then only, that an audience feels that it is in the presence of a reality rather than a fiction. To an 45 audience an ounce of genius has more weight than a ton of talent; for though it respects the later, it reverences the former. But the creative power, divine as it may be, should in common gratitude pay due regard to the reflective; for Art is the handmeid of Genius, and only 50 asks the modest wages of respectful consideration in payment for her valuable services. A splendid torrent of genius ought never to be checked, but it should be ‘wisely guided into the deep channel of the stream, from whose surface it will then reflect Nature without a 55 ripple. Genius dyes the hues thet resemble those of the rainbow; Art fixes the colors that they may stand. In the race for fame purely artificial actors cannot hope to ‘win against those whose geniusis guided by their art; and, on the other hand, Intuition must not complain if, 60 unbridled or with too loose a rein, it stumbles on the course, and so allows a well-ridden hack to distance it a ‘The primary purpose of the passage is to A) issue a warning, B) outline a procedure. ©) describe a puzzling situation. D) examine a controversy. a ‘The author's remark in lines 4-6 is best described as, A) aprediction. B) an exaggeration, ©) aconcession. D) aconjecture 13 | i ‘The passage as a whole suggests that “spontaneity” (line 15) ‘A) isa sign of an organized and artistic mind and helps actors avoid becoming muddled. B) occasionally results in effective performances but cannot be relied upon to do so. ©) isa quality that only actors of genius possess and should be highly valued. D) should be avoided in acting, as it produces only poor quality performances. ‘Which choice provides the best evidence for the answer to the previous question? A) Lines 1-8 (“T have. . . them”) B) Lines 18-20 ("Itis... part”) C) Lines 20-26 (“If he... muddle") ‘D) Lines 44-51 (“Toan.. .. services") As it appears in lines 27-28, the phrase “mysterious friends” most nearly refers to ‘A) genius and detailed planning. B) creativity and the ability to improvise. ©) adequacy and impulsiveness. D) surprise and jubilance, Bese According to the information in the passage, the “impenetrable veil” (line 41) has which of the following, effects? A) Itmakes a performance difficult to understand. B)_Itresultsin a cold and passionless performance. ©) It causes an audience to forget that itis viewing an attificial performance. D) Its the result of an actor leaving too much of his performance to chance. ‘Which choice provides the best evidence forthe ‘answer to the previous question? A) Lines 13-17 (“In the... name") B) Lines 22-26 (“but should ... muddle”) ©) Lines 33-37 (“Acting.... despised”) D) Lines 43-44 (“It is. fiction”) ‘As used nine $2, checked” most realy means A) examined. B) compared, ©) curbed. D) supervised. ‘As used in line 56, “fixes” most nearly means A) repairs. B) plans. C) shades. D) establishes. In a final sentence ofthe passage the author A) predicts an intended outcome, B) concludes with an illustration, ©) summarizes his major points, D) qualifies his central argument. OTPRcécne Meigs e| 27 Answers and Explanations Il. A “The answer is (A). As early as the second sentence, the author warns, this is a dangerous course. In every paragraph, the author tells the reader that preparing for a role is one requiring art and that even genius that comes from spontancous inspiration needs to be guided by art. In line 56, the author states that art fixes the colors that they may stand, While acknowledging the usefulness of genius or inspiration, the author, in the very first sentence, warns against relying on those things alone and not leaving the performance to chance. The answer isn't (B) because the author does not give step-by-step guidance on how to give a good acting performance. The answer isn’t (C) because for the author the answer isn’t puzzling at all, He has very clear ideas on the subject. The answer isn't (D) because to the author the subject is not controversial. He is convinced to the accuracy of his point of view. And, for this reason, he doesn't examine it so much as explain it. 12. © Theanswer is (C). In lines 1-3 the author makes it clear that being émpulsive and leaving things to chance is a dangerous course. In the lines referenced in the question, the author starts the sentence with, J will admit, This signals that he will present an exception to what has previously said. He goes on to write that under such conditions great effects have been produced, The answer isn't (A) because he isn't predicting; he’s describing events from the past. The answer isnt (B) because he's very clear that he is describing things as he as seen them. “The answer isn’t (D) because he is talking about performances he has seen, not ones he imagines he may see in the facture 13. B___Theanswer is (B). The author is very clear that spontaneity isnot tobe commanded in lines 16-17. You cant rely on its happening. The answer isnt (A) because the auchor is clear that actors who reply on it are not organized. He is clear that actors that do rely on being spontaneous run the risk of being in a maze and his audi- ence in a muddle (lines 25-26). The answer ise’ (C). The author doesn't divide actors into those of genius who can rely on inspiration and those who do nor have genius and must rely on inspiration. He writes that all actors need to prepare carefully so that if inspiration or genius does appear, that force can be best utilized and, if genius and inspiration do not appear, the actor can still give a competent performance. Answer (D) is directly contradicted by the passage more than once. For example, in lines 44-45 the author states that to an audience an ounce of genius dhas more weight than x ton of talent 14. A Choice (A) provides the best support for the previous answer. In these lines, the author concedes that rhere are many instances where great effects have been produced that were entirely spontaneous. Ever: though he docs not believe this is the best approach to acting, he admits it can yield good results sometimes. The answer is not (B), (C), or (D) because in all of them, the author asserts his overall point that actors need to have good peeformance outlines instead of relying on inspiration. 15. B__Theanswer is (B). In lines 14-17 the author refers to spontaneity and inspination as qualities that may or may not decide to come to an actor during a performance. Therefore, these are the mysterious friends mentioned in the lines in question. Choice (A) is half-right because genius is correct. But remember, half-right ‘means all wrong, Choices (C) and (D) use deceptive language; both use pairs of words randomly but do not, when combined, describe the inspiration the author discusses. 16. C This paragraph begins with che author describing the importance of getting the technical as- pects of a performance right but admitting that just being technically good is not enough. To make a great per- formance, an «ctor covers over the technical aspects with the invisible vel of genius that can convince the audience that what they are seeing is not fiction but reality ‘The right answer isn’t (A) since the veil causes the audience to fully connect with the performance. Ic isn't (B) because the veil results in the warmth of pasion in tragedy and the sudden glow of humor in comedy. Ic ise’ (D) because the accor has given che performance with care. 17. D___Theauthor believes an actor must prepare a role in a technical way so that, ifhe is not inspired, hell stil give a good performance. Ihe relies only on being inspired, his performance may be great one day but not even deceat the next day. So, the author believes if an actor prepares as well as he can technically and then becomes inspired ata given performance, the result will be genius. Under those conditions the audience feels it isin the presence ofa reality rather than afetion as staed in lines 43-44. The answer isnt (A), (B), or (C) because those answer choices don't reference both parts of good acting. 18 C__Incontext, checked must mean limited, because the author is saying that genius must not be lim- ited but instead wisely guided. Choices (A) and (B) do not match this meaning. Choice (D) is incorrect because if the genius is not supervised, then it cant be wisely guided. This leaves the right answer, (C).. 19. D__In context the word must mean cause to stand ot stay in place, as in the colors need to stay in place. The only choice chat matches this meaning is (D). Choices (A) and (B) do not match this meaning. Choice (C) may be tempting because one can shade colors. But it doesn't not make sense in context, s0 (C) is also not 20. B_ The author concludes with a description of a horse race, in which the beter-prepared rider ‘may outpace the more talented, but less prepared rider. This isan illustration of the author’s central argument that preparation is more important than waiting for inspiration. Choice (B) best capcutes the author's intention. Choice (A) is incorrect, because the author is not predicting an outcome fora specific event, nor would any actor ever intend to put on 2 bad performance. ‘The author illustrates his major point, but he does not summarize all his points, so (C) is incorrect. Choice (D) is incorrect because the author is not qualifying his central argument; he is restatingit. 10 » 2 30 3s 45 This passage was adapted from a speech given by Susan B ‘Anthony in 1872 after she was arrested for voting inthe 1872 Presidential Election, Friends and fellow citizens; I stand before you tonight under indictment for the alleged crime of having voted at the last presidential election, without having a lawful right to vote. It shall be my work this, evening to prove to you that in thus doing, I not only committed no crime, but, instead, simply exercised my citizen's rights, guaranteed to me and all United States citizens by the Constitution, beyond the power of any State to deny. Our democratic-republican government is based on the idea of the natural right of every individual member thereof to a voice and a vote in making and executing the laws. We assert the province of government to be to secure the people in the enjoyment of their inalienable right. We throw to the winds the old dogma that government can give rights. No one denies that before governments were organized each individual possessed the right to protect his own life, liberty and property. When 100 to 1,000,000 people enter into a free government, they do not barter away their natural rights; they simply pledge themselves to protect each other in the enjoyment of them through prescribed judicial and legislative tribunals. They agree to abandon the methods of brute force in the adjustment of their differences and adopt those of civilization. ‘The Declaration of Independence, the United States, Constitution, the constitutions of the several states, and the organic laws of the territories, all alike propose to protect the people in the exercise of their God-given rights. Not one ofthem pretends to bestow rights. ‘The preamble of the Constitution says: ‘We, the people of the United States, in order to form. ‘a more perfect union, establish justice, insure domestic tranquility, provide for the common defense, promote the general welfare, and secure the blessings of liberty to ourselves and our posterity, do ordain and establish this Constitution for the United States of America. It was we, the peoples not we, the white male citizens; nor we, the male citizens; but we, the whole people, who formed the Union. And we formed it, not to give the blessings of liberty, but to secure them; not to the half of ourselves and the half of our posterity, but to the whole people—women as well as men. And itis a downright mockery to talk to women of their ‘enjoyment of the blessings of liberty while they are denied the use of the only means of securing them, so 55 as 0 provided by this democratic-republican government— the ballot. Charles Sumner, in his brave protests against the Fourteenth and Fifteenth Amendments, insisted that as soon as by the Thirteenth Amendment the slaves became free men, the original powers of the United States Constitution guaranteed to them equal rights - the right to vote and to be voted for. When, in 1871, asked that senator to declare the power of the United States Constitution to protect women in their right to vote - as he had done for black men - he handed me ‘a copy of al his speeches during that reconstruction period, and said: ut “sex” where I have “race” or “colon” and you have here the best and strongest argument I can make for woman. There is not a doubt but women have the constitutional right to vote, and I will never vote for a Sixteenth Amendment to guarantee it to them. 1 voted for both the Fourteenth and Fifteenth under protest; would have insisted tha: the power of the original Constitution to protectall citizens in the equal enjoyment of their rights should have been vindicated through the courts. But, friends, when in accordance with Senator Sumner’s counsel I went to the ballot-box last November, and exercised my citizen’ right to vote, the courts did not wait for me to appeal to them—they appealed to me, and indicted me on the charge of having voted illegally. ‘Though the words “persons! “people?” “inhabitants?” “electors, “citizens” are all used indiscriminately in the national and state constitutions, there was always a conflict of opinion, prior to the war, as to whether they were synonymous terms, but whatever room there was for doubt, under the old regime, the adoption of the Fourteenth Amendment settled that question forever in its first sentence: All persons born or naturalized in the United States, and subject to the jurisdiction thereof, are citizens of the United States, and of the state wherein they reside. ‘The only question left to be settled now is: Are ‘women persons? And I hardly believe any of our ‘opponents will have the hardihood to say they are not. Being persons, then, women are citizens; and no state has a right to make any law, or fo enforce any old law, that shall abridge their privileges or immunities. Woman's Sutags BotoreRatleaton ofthe 10m Amendment 1820, Leek aa (Patt womans stg by 10 1 Wowemun tego by 190 ‘This graphic, created by Sandra Opdycke in 2000 for the The Rutledge Historical Alas of Women in America, depicts ‘women's suffrage in the United States before the passage of the Nineteenth Amendment in 1920, which guaranteed womens right to vote, What is the primary purpose of this passage? A) Toargue for a new amendment granting women the right to vote B) To provide a legal defense for the author’ ‘upcoming trial ©) Todemonstrate that women already have the right to vote D) To encourage voter to elect Charles Sumner As used in line 14, “enjoyment” most nearly means A) use. B) pleasure. ©) satisfaction. D) gratification. Which of the following best describes the role of laws in regards to natural rights? A) When people make a government they give up their natural rights by making laws B) A government makes laws in order to grant natural rights to its citizens. C) Judicial and legislative tribunals decide on and confer the natural rights of citizens D) Itis through laws that a government protects citizens’ preexistent natural righ's. ‘Which choice provides the best evidence for the answer to the previous question? A) Lines 4-9 ("Itshall ... deny”) B) Lines 26-30 (“The Declaration .. bestow rights”) ©) Lines 38-40 (“It was... Union”) D) Lines 84-86 (“All . reside”) ‘The author mentions the preamble of the Constitution (lines 31-37) primarily in order to A) provide a precedent for her argument for gender equality B) illustrate the degree to which the Constitution clearly does not grant women certain liberties ©) argue for a complete revision to she Constitution and Bill of Rights. D) explain the bill for which she was voting when arrested, Which choice provides the best evidence for the answer to the previous question? A) Lines 16-19 ("No one... proper'y”) B) Lines 23-25 (“They... ©) Lines 38-40 (“It was we... . Union”) D) Lines 43-48 (‘And itis... the ballot”) civilization’) erPRe catia tens Le | 31 Based on the information in the passage, Sumner protested the Fifteenth Amendment, which prohibits denying the right to vote based on race, because 'A)_he did not believe that former slaves deserved the right to vote B) he thought it was redundant with the Constitution, ©) he predicted that it would be vetoed. D) he had been counseled by Anthony to do so. ey ‘Asused in line 82, “settled” most nearly means A) immigrated. B) paid. ©) silenced. D) decided. ‘The primary purpose of lines 84-86 as they relate to the rest of the passage is to A) provide evidence for an earlier claim. B) offer a position that the author immediately disputes. © illustrate the difficulties of interpreting, constitutional law. D) explain the core of a proposed piece of legislation. 32 | erPastuacan hain ‘The principal rhetorical effect of the question in lines 87-88 is to ‘A) highlight the obviousness of an answer in the affirmative. B) begin a genuine dialogue on the legal definition of “person” ©) demonstrate the absolute necessity of a new voting rights law. D) suggest a topic for a future political speech. Which claim about women’s suffrage is supported by the graphic? |A) After the passages of the Thirteenth Amendment, ‘American women in all states had no barriers to voting. B) Anthony could have avoided jail had she traveled to Colorado (CO) to vote in 1872 ©) Until 1920, the state governments of Alabama (AL) and Georgia (GA) didn't believe women were people. 1D) Some states allowed women to vote even before the 19th amendment guaranteed this right, THIS PAGE IS LEFT INTENTIONALLY BLANK. oP ween eign | 38 ™ Answers and Explanations 21. © Theauthor's primary argument is thac women’s right to vote has already been established in the Const to vote is already established in the Consticution, so eliminate (A). The author's upcoming tial was no: discussed, jon. Therefore (C) is the best answer. ‘The author does not argue for a new amendment, as women's right so eliminate (B). The election of Charles Sumner was never discussed, so eliminate (D).. 22. A _In this sentence the author discusses how the citizens of a nation protect each other's natural rights and how these citizens choose to exercise those rights. The word use is most similar to how citizens exercise their rights, so (A) is the best answer. Choices (B), (C), and (D) are all synonyms to the word enjoyment, but these answers do not reflect how the word is used in the context of the passage. 23. D___ Inlines 26-30 the passage states, The Declaration of Independence, the United States Constitution, ‘the constitutions of the several states and the organic laws of the territories, all alike propose to prozect the people in the exercise of their God-gwven rights. Not one of them pretends to bestow rights. In other words, the role of laws isto protect a citizen's natural rights. Thus (D) is the best answer. ‘The passage does not discuss giving up natural rights ‘when a government makes laws, so eliminate (A). Choice (B) is the opposite of what the passage says. The passage does not state that the legislative and judicial branches bestow these natural tights, so eliminate (C). 24. B__Theanswer to the previous question indications that the role of laws is not to actually provide a citizen's nacural rights, but to protect these rights. Choice (A) discusses certain rights guaranteed by the Constitu- tion while choices (C) and (D) discuss certain classes of people who should have these rights. Choice (B) discusses ing these rights, Choice (B) is the only choice to provide adequate support for the previous question, so it is che correct choice this concept for pro 25. A Theauthor mentions the preamble of the Constitution, then continues her argument that this preamble refers co people of the United Seates in general, not a specific gender. Choice (A) most accurstely reflects the purpose of the author's mention of this piece of the Constitution. Choice (B) is the opposite of the author's or the Bill of Rights: she claims that women’s, right to vote is already established in these documents. ‘The bill the author was voring for when she was arrested argument. The author does not argue to revise the Const was not mentioned, so eliminate (D). 26. The author quotes the preamble of the Constitution, then continues her argument that this preamble refers to people of the United Sates in general, not a specific gender. ‘This can be seen in lines 38-40. Choice (C) quotes the best evidence from the passage to help answer the previous question. Choices (A), (B), and (D) do not provide eridence of why the author mentions the preamble of the Constitution, 27. B___ The Fifteenth Amendment prohibits denying the right co vore based on race, Sumner protests this Amendment because he argues thac the Constitution establishes the right to vote for all people in the Unites States. Therefore the Fifteenth Amendment is redundant. Choice (B) is the best answer. Choice (A) is the oppo- site of what the passage states. The passage does nor state chat Sumner thought the Amendment would be vetoed ‘or that Anthony counseled him to protest the Amendment, so eliminate (C) and (D). 28. _D__ The passage discusses that even though there may have been a disagreement on what the Con- stitution meart by the words people and citizens, any doubt on the meanings of these words was clarified wih the Fourteenth Amendment, Choice (D) most accurately represents this meaning of clarifying the meaning of these words. Choice (A), (B), and (C) are all synonyms to the word settled, but these words do not reflect the meaning of settled in the context of the passage. 29. A This quote of the Fourteenth Amendment states that all people born in the United States are citizens of that country. The author makes the argument thac the words people and citizens are interchangeable, s0 (A) best reflects why the author mentions the Fourteenth Amendment. The author does not dispute that people born in the United States are citizens of that country, so eliminate (B). The author does not clzim that interpret ing constituticnal law is difficult, so eliminate (C). The purpose of quoting the Fourteenth Amendment was not to explain a proposed piece of legislation—ic was to support the author's previous argument. Eliminate (D). 30. A Tn lines 87-89, che passage asks the herorical question, Are women persons? "The passage contin- ues to answer this question in the next sentence, I hardly believe any of our opponents will have the hardihood to say they are not, The author asks this thetorical question to suggest that itis obvious that women ate indeed persons, 0 (A) is the best answer. The author does not use this rhetorical question to start a dialogue on the legal definition of the word pason; the author is claiming this definition obviously includes women. Eliminate (B). The author does not argue for a new voring rights law because she believes women’ right ro vore is already established in the Constitution, Eliminate (C). The passage does not mention future political speeches, so eliminate (D). 31. D__ The graphic shows that several states, for example Utah and Colorado, permitted women the tight to vote before the Nineteenth Amendment was passed in 1920. Therefore (D) is supported by the graphic. Choice (A) is incorrect because the graph shows that many states had did not allow women to vote in 1919, so the Thireeenth Amendment did not guarantee suffrage forall women. Colorado did not allow women to vote until 1893, so Anthony would still not have been able to vore there in 1872. Choice (B) can therefore be eliminated. Though states like Alabama and Georgia did not allow women to vote until 1920, there is no evidence in the graphic that this means they didn't believe them to be people. So (C) is incorrect. erences dns | 35 x ‘This passage is adapted from The Federalist Papers, a series of essays and articles jointly written by Alexander Hamilton, John Jay, and James Madcon, all of whom were Founding Fathers (of the United States John Jay wrote the essay below, Federalist, oper 2, on October31, 1787. In ithe discusses the question of maintaining the United States asa single nation or dividing the country into a collection of small independent sovereignties. Jay's essay was written in the context of rising concern among, ‘Americans that one or more European powers would attempt to return the UnitedStates to its former status as a colony. ‘To the People ofthe State of New York: WHEN the people of America reflect that they are now called upon to decide a question, which, in its tine consequences, must prove one of the most important 5 that ever engaged their attention, the propriety oftheir taking a very comprehensive, as well as a very setious, view of it, will be evident. Nothing is more certain than the indispensable necessity of government, and itis equally undeniable, 10 that whenever and however itis instituted, the people rust cede to it some of their natural rights in order to vestit with requisite powers. It is well worthy of consideration therefore, whether it would conduce more to the interest of the people of America that they 15 should, to all general purposes, be one nation, under one federal goverament, or that they should divide themselves into separate confederacies, and give to the head of each the same kind of powers which they are advised to place in one national government. 20 Ithas until lately been a received and uncontradicted opinion that the prosperity of the people of America depended on their continuing firmly united, and the ‘wishes, prayers, and efforts of our best and wisest citizens have been constantly directed to that object. 25 But politicians now appear, who insist that this opinion is erroneous, and that instead of looking for safety and happiness in union, we ought to seek it in a division of the States into distinct confederacies or sovereignties. However extraordinary this new doctrine may appear, 30 it nevertheless has its advocates; and certain characters ‘who were much epposed to it formerly, are at present of the number. Whatever may be the arguments or inducements which have wrought this change in the sentiments and declarations of these gentlemen, it 45 certainly would not be wise in the people at large to adopt these new political tenets without being fully convinced that they are founded in truth and sound policy. Ithas often given me pleasure to observe that 40 independent America was not composed of detached and distant territories, but that ene connected, fertile, wide-spreading country was the portion of our western sons of liberty. Providence has in a particular manner blessed it with a variety of soils 45 and productions, and watered it with innumerable streams, for the delight and accommodation of its inhabitants. A succession of navigable waters forms a kkind of chain round its borders, as if to bind it togethers ‘while the most noble rivers in the world, running at 40 convenient distances, present them with highways for the easy communication of friendly aids, and the mutual transportation and exchange of their various commodities. With equal pleasure I have as often taken notice that Providence has been pleased to give this 55 one connected country to one united people—a people descended from the same ances‘ors, speaking the same language, professing the same religion, attached to the same principles of government, very similar in their ‘manners and customs, and who by their joint counsels, 60 arms, and efforts, fighting side by side throughout a long and bloody war, have nobly established general liberty and independence. ‘This country and this people seem to have been made for each other, and it appears as if it was the design of Providence, that an inheritance so proper and convenient for a band of brethren, united to each other by the strongest ties, should never be split into a number of unsocial, ealous, and alien sovereignties.. é oe ‘The position that Jay takes in his essay can best be described as that of A) an impartial onlooker pointing out both sides of ai issue. B) aleader arguing for a particular outcome, (©) an intellectual studying a historical decision, D) a spokesperson seeking a reasonable compromise. As used in line 12, “vest” most nearly means A) clothe, B) endow. ©) abandon. D) belong. Meee Based on the information in lines 32-38, Jay believes that those who choose to support dividing the country into separate confederacies A) are contradicting the wishes, prayers, and efforts of the majority of its citizens. B) should not do so without first examining the issue thoroughly, ) value independence over safety and happiness. D) are threatening the prosperity of the nation by encouraging political turmoil. ‘Which choice provides the best evidence for the answer to the previous question? ‘A) Lines 20-24 (“It has... . object”) B) Lines 34-38 (“It certainly... policy”) C) Lines 39-43 (“It has... liberty”) D) Lines 53-5 (“With ... people”) In lines 26-28 (“instead of .. .or sovereignties”), what is the most likely reason that Jay introduces the idea of a “division of the States”? |A) To disagree with the claim that people must give up some of their natural rights to the government B) To introduce a point of view that he will ater counter by showing that the people of the country are linked by strong bonds ©) Toargue that dividing the states into separate sovereignties would result in increased prosperity for citizens D) To contend that politicians have been hesitant to adopt the doctrine of division because they find the concept too astonishing answer to the previous question? |A) Lines 8-12 (“Nothing B) Lines 25-28 (“But ©) Lines 29-32 (“However ... number”) D) Lines 63-68 (“This .. sovereigntis”) power”) overeignties’) As used in line 33, “wrought” most nearly means A) caused, B) ornamented, ©) finished. D) engraved. In ines 39-43 Jay makes a distinction between A) self-reliance and liberty. B) secluded provinces and a unified dominion. ©) deserted marketplaces and lush farmland. D) division and independence. epee ae wae Based on the information in the fourth paragraph, ‘waterways play all of the following roles in America EXCEPT ‘A)_actas boundaries that encircle the nation. B) facilitate trade. ©) create channels for settlements toexchange messages. D) provide sources of drinkable water. corPReaatee haley. Ue | 37 ‘The principal rhetorical purpose of the phrases in lines 55-58 (“a people descended ... principles of government”) isto A) Discuss four reasons that show the impossibility of dividing the United States into distinct confederacies. B) Suggest a four-part method of maintaining the United Statesas a single nation. ©) Argue against division by emphasizing the ways in ‘which the people of the country are united. D) Show that Jay believes that the people of the United States all came from the same families, grew up speaking the same languages, and developed the ‘same beliefs. a In the final paragraph of the passage, Jay A) predicts a probable development. B) states his central argument. ©) summarizes by introducing a metaphor. D) concludes by qualifying a previous statement THIS PAGE IS LEFT INTENTIONALLY BLANK. Answers and Explanations 11. B__ Theblurb at the beginning of the passage indicates that Jay is one of the Founding Fathers of the United States, so he is not an impartial onlooker. He also specifically argues that the United States should remain as a single country, rather than dividing into smaller, separate nations, so he is not primarily concerned with pointing out both sides of the issue. Thus, (A) is incorrect. Since Jay is a Founding Father of the United States, he can appropriately be described as a leader. In the final paragraph of the essay he states that the United. States should never be split into a number of unsocial, jealous, and alien sovercignties, He is therefore arguing for a particular outcome—that the United States remains a single country rather than a collection of independent smaller countries. Choice (B) is thus the correct answer. According to the blurb, Jay wrote his essay in 1787, while United States citizens were still debating whether to remain as a single nation or divide into separate nations, so he was not discussing a historical decision; he was discussing an issue current in his time. Choice (C) is therefore Jay argues for a particular position ~ that the United States should never be split into a number of .. sovereigaties—he is not seeking a compromise. Thus, (D) is not the correct answer incorrect. Finally, s 12, B__ Inthesentence in question, Jay says that the people must cede, or give up, some of their natural rights to the government. He then says that they must give up these rights int order to vers it with requisite powers. Since people are givirg their rights to the government, they must be giving the government power. ‘Thus, a good ‘word to put in place of the word vest might be give. Since clothe, abandon, and belong all have meanings that are different from the meaning of the word give, you can eliminate (A), (C), and (D). Since endow means give, (B) is the correct answer. 13. B__Jaydoes mention the wishes, prayers, and efforts of some of the citizens. However, before that he begins the paragraph by saying that ir has until lately been a received and uncontradicted opinion that the prosperity of the people of America depended on their continuing firmly united. The words until lately indicate that the situation has changed. Therefore, there is no evidence that the majority of the people, a the time at which Jay wrote, were against dividing che country. When he spoke of the wishes, prayers, and efforts of the people, he was speaking about their former, rather than their present feelings and actions. Therefore, (A) is incorrect. At the end of the third paragraph, Jay states that it certainly would not be wise inthe people at large to adopt hese new political tenets without being fully convinced that they are founded in truth and sound policy Therefore, he believes that penple should examine the facts closely before deciding to support division, and (B) is the correct answer. Jay does says that some insise that instead of looking for safety and happiness in union, we ought to seek it in a division of the States. However, the passage does not indicate that those individuals value independence over happiness and safety, only that they believe that they should seek happiness and safety through a different method than remaining united. “Thus, you can eliminate (C). Finally, while the passage does indicate that the prosperity of the people will not best be served by dividing the nation into smaller confederacies, Jay does not indicate that the reason those individuals threaten the nation’s prosperity is that they encourage political turmoil. ‘Therefore, (D) is incorrect. 14, B___ The answer to the previous question indicates that Jay cautions those who would divide the country to first examine the issue thoroughly. Answer choice (A) discusses a widely held belief about the wishes of the country, and so is therefore incorrect, In choice (B), Jay urges people considering division :o make sure their arguments are rooted in truth and sound policy. This provides evidence forthe previous question, and so choice (B) is correct. Answer choices (C) and (D) discuss the unity of the country by geography and by people, and so ate incorrect. 15. B___Jay does mention chat she people must cede to it some of their natural rights co the government. However, immediately before he makes this statement he says that this fact is undeniable, so he is not crying to disagree with this claim, and (A) is incorrect. The sentence in question states that politician: now appear, who insist chat. ..we ought to seek init a division of the sates into distinct confederacies. Note that this sentence provides the politicians point of view, rather than Jay's. At the end of the essay, Jay gives his own opinion, saying that the country should never be split into a number of unsocial, jealous, and alien sovereignties. Jay therefore disagrees with the politicians. In the second to last paragraph, he lists a number of factors that the people of the United States hhave in common, and in the final paragraph he states that the people are united to each other by the strongest ies. “Thus, he counters, or argues against the claim that the nation should be divided, and he docs this by demonstrat- ing that the people of the country are linked by strong bonds. Choice (B) is therefore the correct answer. The politicians believe that the country should be divided, but Jay does not, so (C) goes against Jay’s argument and is incorrect. The second paragraph indicates that many politicians support the idea of division, so they are not astonished by it. Choice (D) is therefore incorrect. 16. “The statement in (A) is simply a fact that Jay says is undeniable, and does no: relate to the ea- son that Jay mentions the opinion of the politicians, who state chat the country should be divided into distiner confederacies. Thus, (A) is incorrect. Choice (B) references the lines referred to in question 14, but question 14 asks not what those lines say, but the reason thar Jay uses those lines. In order to discover why Jay uses those lines, you must examine how they relate to other parts of the passage, Therefore, these lines in themselves do not answer the question, and (B) is incorrect. The lines referenced in (C) indicate that many people support the idea of division, but they do not explain why Jay introduces the idea of division in the previous sentence, so (C) is incorrect. Choice (D) provides the reference that proves that (B) from question 14 is the comect answer: these lines indicate that Jay does not suppore the idea of division because the people of the country are outed to each other bythe strongest tes, should never be split into a number of unsocial, jealous, and alien sovereignie. Since this is Jay's point of view, he must have introduced the point of view of the politicians—the point of view that division would be beneficial in order to counter that point of view. Thus, (D) isthe correct answer. orPnetten ting. | AL 17. A__ Immediately before the sentence in question, Jay states, However extraordinary this new doctrine may appear, it nevertheles has its advocates. This suggests that he finds the new doctrine startling, and indicates, that he may be surprised that so many people support i. In the next sentence, he uses the phrase, Whatever may be the arguments or inducements, suggesting that he is discussing the reasons that many people suppart che new doctrine, and that these reasons are either puzzling or varied. Immediately after the word wrought he goes on to talk about the change in sentiments so it is reasonable to assume that he is discussing the reasons for the change, or in other words, what caused so many people to begin supporting the new doctrine. Thus, a good word to put in place of the word wrought might be caused. Putting ornamented, finished, or engraved in place of wrayght in the passage would not make sense, so you can eliminate (B), (C), and (D). The correct answer is (A). 18. B__Jaydaes mention both independence, which is similar to self-reliance, and diberty in the lines ref crenced. However, he does not make a distinction between these two concepts, so (A) is incorrect. The distinction he makes is between detached and distant territories, which can be said to be secluded provinces, and one connected, fertile, wide-spreading country, which can be said to be a unified dominion. ‘Thus, (B) is the correct answer. Jay does not mention marketplaces anywhere in these lines, so (C) is incorrect. He does discussion the idea of division throughout the passage, but he does not compare division with independence, so (D) is also incorrect. 19. D__Inlines 47-48 Jay states that « succession of navigable waters forms a kind of chain round its bor- ders, as if to bind it tether. This indicates that waterways act as boundaties that encircle the nation, so you can climinate (A). In lines 49-53 Jay states that the most noble rivers in the world... present them with highways for. exchange of their commodities. Thus, the passage indicates that rivers facilitate trade, so you can eliminate (B). In lines 50-51, Jay states that the rivers create highways for the easy communication of friendly aids. Thus, rivers cxe- ate channels for exchanging messages, ot communications, so (C) is incorrect. The passage makes no mention of drinking water, so (D] is the correct answer. 20. © Choice (A) discusses she impossibility of dividing the U ever, Jay does not state that dividing the countries into separate countries is impossible—only that the country d States into separate countries. How- should never be split—so (A) goes beyond the passage and is therefore incorrect. Jay mentions that the people of the United States are a people descended from the same ancestors, speaking the same language, professing the same religion, astached 10 the same principles of government, However, these are facts that are already crue abour the people, rather than methods of keeping the people together, so (B) is incorrect. Immediately before the phrases mentioned in the question, Jay states that Providence has been pleased to give this one connected country to one united people, He then goes on to list the ways in which the people are united. Thus, the purpose of the phrases in question must be to emphasize the ways in which the people are united, so (C) is the correct answer. In the part of the passage in question, Jay does state that the people of the United States are a people descended from the same ancestors, speaking the same language, professing the seme religion, attached to the same principles of government. “This statement is similar to the statement that the people of the United Stats all came from the same families, grew up speaking the same languages, and developed the same belief. However, the question asks, not what Jay states in the passage, but the ractorical purpose of the lines, or in other words, why Jay makes his statement. The reason thar he makes the statement in question is to emphasize that Providence has been pleased to give this one connected country o one united people, Therefore, (D) docs not accurately describe the purpose of the lines. Nate too (D) goes slightly beyond the passage, since che people of the United States could profess the same religion and be at- tached to the same principles of government, but still have varying beliefs about other issues. 21, B__Jaydoes not make any predictions in the final paragraph, so you can climinate (A). However, in this paragraph he does finally give his opinion on the issue of whether the United States should be spe into sepa rate confederacies; he states clearly chat this country...should never be split into a number of unsocial, jealous, and alien sovereignies. Therefor, in the final paragraph, Jay states his central argument, so (B) is the correct answer, He does not irtroduce any metaphors in this paragraph, so (C) is incorrect, and he does not qualify any previous starements, 80 (D) is also incorrect. cermmeseaeetP dg | 8 This passage is adapted from Robert M. Edsel, The Monuments ‘Men: Allied Heroes, Nazi Thieves, and the Greatest Treasure Hunt in History. © 2009 by Robert M, Edsel. The passage describes the setting ofa salt mine in the Austrian Alps. From there, the road begins a climb so steep that the Patschen Pass seems a mild slope by comparison. Along the road runs a clear, crashing alpine stream, and. tine beyond are the immense and breathtaking mountains. 5 They are limestone deposits, formed in the depths of an ancient sea, and even on the sunniest day they are pale gray beneath their caps of snow. A bleak stone building, perched precariously above a thousand-foot precipice, ‘marks the beginning of the end, Beyond is only a low 10 irregular building and a wall of rock, the steep side of the Sandling Mountain, Bored into the mountain is @ small tunnel, the main entrance to an ancient salt mine. Local legend holds that salt had been mined here for three thousand years—before the founding of Rome, at 15 the height of the encient Egyptian empire. Local written records, however, only date back to the 1100s, In those days, at the turn ofthe first millennium, salt was one of the foundations of civilization. Without it, food couldn't be preserved or transported, so whole 20 societies survived because of salt. Roman legionnaires ‘were sometimes paid in salt (the basis of the English word “salary”), and merchants trod the salt roads in large caravans, lirking the Western world of Europe with the Eastern world of Asia and Arabia. In Tibet, 25 Marco Polo noticed that salt was pressed into wafers, imprinted with the image of the Grand Khan, and used as money. Timbuitu, the great lost civilization of Africa, valued salt as highly as gold. The early Germans, whose Visigoth ancestors sacked Rome and threw civilization 40 into darkness, were economically dependent on their salt mines, and especially the taxes for their salt trading routes. The city of Munich, an early base of power for, the Nazi Party, was founded in 1158 so the ruler of Bavaria could more easily collect a tax on the salt being 35 transported from the city of Salzburg (German for “Salt Castle”) ‘And throughout the centuries, as cities and empires rose and fell, the Steinberg mine in the Sandling ‘Mountain of Austria just above the village and lake 40 known as Altaussee, continued to produce salt. The salt ‘was not mined with picks and shovels, but dissolved by the flow of water through special pipes and sluices, The water came from the mountain above, especially during. the spring snowmelt, and descended by gravitational 4 force through the mine, There it was inundated with 44 | cormntinercniP ergs ie rock salt, then sent down the mountain to Bad Ischl, ‘more than seventeen miles away, where the brine was evaporated to form pure crystal salt. It was left to 125 miners to maintain the pipes and sluices, shore up the 50 catacombs against the pressure of the mountain, and ‘make sure the vast labyrinth of rooms and tunnels didn't merge together and destabilize the entire structure. Since the 1300s, this job had been performed by ‘members of a small group of families, all living in the 55 hills near the mine. Over the centuries humans grew larger, but the miners stayed the same size, until they eventually seemed dwarfed by the demands of the mine and their time underground (diet and inbreeding were ‘more likely causes). Even in the early twentieth century, 60 this small isolated community spoke a dialect last popular in the Middle Ages. They explored their tunnels with acetylene torches, and wore the white linen suits and peaked caps of medieval miners. But in the winter of 1943-1944, the salt mine at 465 Altaussee was assaulted by the modern world. First came the tracked vehicles necessary for maneuvering over the roads in the winter, when the five meters of snow were almost level with the treetops. They were followed by supply jeeps, and eventually a seemingly 70 endless line of Nazi officers descended on the mine as guards, Workers arrived, expanding catacombs and building wooden floors, walls, end ceilings in dozens of salt chambers. Giant wooden recks were assembled in ‘workrooms deep within the mcuntain and hammered 75 into position, in some places three stories high. Experts and clerks moved ins a shop was built deep inside the mine where technicians could work and even live for days ata time, And it was all dene for art. Which of the following best expresses the main idea of the passage? ‘A) The salt mines in Altaussee have a long and varied history. B)_Salthas been an essential component of civilization since the Roman Empire. ©). Mining techniques in Altaassee were not ‘modernized until the Nazis arrived. D) ‘The presence of the salt mines made Germany into ‘a modern world power. ‘The author uses the word “precariously” (line 8) in order to ‘A) suggest that the sone building is in danger of. falling down the diff B) describe the rugged and remote terrain around the salt mine. ©) imply that the buildings associated with the salt mine were poorly constructed. D) indicate that the people who built the stone building were skilled mountaineers. 1 | i ‘As used in line 15, “height” most nearly means A) pinnacle. B) epitome. ©) elevation. D) eminence, ‘The author most likely includes the description of the importance of salt through history in the second paragraph in order to ‘A) share some interesting but ultimately irrelevant facts B) suggest that the Nazi party would not exist without the salt tax. ©) emphasize the great value of salt in many cultures. D) provide historical context for the Steinberg mine. 125 | ‘The passage indicates that Sanding Mountain is ‘A) asale mine first established by Egyptians. B) a milder slope than Pétschen Pass. ©) the remnant of an ancient sea bed. D) ableak and boring place. ei a Se ‘As used in line 49, “shore” most nearly means A) river bank. ‘B) land. C) buttress. ‘D) beam. "The passage most strongly suggests which of the following about the Steinberg Mine? ‘A)_Itwas owned by the same family for many centuries until the Nazis came. B) Itused a unique system to mine salt with water instead of picks and shovels. ©) twas inaccessible during the winter before the Nazis brought their tracked vehicles. D) Itwas mostly unaffected by political and social upheaval through the ages. Which choice provides the best evidence for the answer to the previous question? A) Lines 37-40 (‘And throughout... salt”) B) Lines 40-42 (“The salt... sluices") ©) Lines 53-55 (“Since the 1300s, ... the mine”) D) Lines 65-68 (“First came... treetops") a § In the passage, the author draws a contrast between A) medieval and modern mining techniques. B) local mining families and Nazi officers. ©) supply jeeps and mine carts D) coal mining and salt mining, Eee Which choice provides the best evidence for the answer to the previous question? A) Lines 61-63 ("They ... minors”) B) Lines 64-65 (‘But ... world”) ©) Lines 68-71 ("They.... guards”) D) Lines 73-75 (Giant... high”) Lo TREsacten Hl UE In the context of the passage, the author's use of the phrase “And it was all done for art” (line 78) is primarily meant to convey the idea that A) the Nazis carved sculptures into the salt mines in Altaussee, B) modern mining techniques were seen by the Nazis as an art form. ©) the Nazis took over the salt mines for a purpose other than mining. D) the salt mines were turned into a mining museum, by the Nazis. THIS PAGE IS LEFT INTENTIONALLY BLANK. orPnesucntan angele | 7 Answers and Explanations 21. A The majority of the passage details salt mines, their locations and histories, Use POE to find an answer that best matches this. Choice (A) is a good fit, so keep it. Salt’s use since the Roman Empire is mentioned in the third paragraph, but it not the main idea, so eliminate this answer. Choice (C) is never mentioned, so get rid of it. Eliminate (D) as well since i's only really discussed in the second paragraph; furthermore, the author never says that the salt mines made German into a modern world power. Choice (A) is the best answer. 22. B_—_Theauthor’s purpose in using the word precariously in this sentence is to provide context for the reader about the area where this bleak stone building is located. The author uses descriptive words like perched and «a thousand-foot precipice in this sentence, so this building is siting, almost like a nest in a tree, at the very top of an extremely steep cliff. The author then mentions other aspects of the land surrounding the salt mine: the steep side of the Sandling Mountain and the entrance of the salt mine is bored into the mountain, Choice (B) is a good march, so keep it. There is nothing in the text to support that the building “is in danger of falling down the [Cl), or the builders were “skilled mountaineers” (choice {D)). All of those answers would require making assumptions beyond the scope of the text, so they should clif” (choice [A]), the mines “were poorly constructed” (chi be eliminated. Choice (B) is the best answer. 23. A ‘Cross off the word height and use the context to choose your own word to fit. The sentence states before the founding of Rome, so the best replacement for height could be best part. Choice (A) is a good match, so keep it. Choice (B) means best example, which doesn't quite fit, so get tid of it. Eliminate (C) because it’s too similar to the actual definition of height. Also, eliminace (D) because it means high official. Choice (A) is the best 24. D__According to the blurb, this passage describes the setting ofa salt mine in she Austrian Alps, ‘There- fore, the information provided in the second paragraph should connect back ro this topic and main idea of the passage (che history of the salt mines in Altaussee). By including information about the importance of salt throughout history, the author informs the reader about the value of the mines existence, which makes (D) a good option. Be careful with (C). This question is not asking what the author is discussing in this paragraph. This question is asking why the author has included this information, so eliminate (C). Choice (A) is incorrect because the information is relevant ro the passage, and (B) is incorrect because there is no information in the text to sup- port this conclusion nor is this choice explaining why the author describes the importance of salt. Choice (D) is the best answer. 25. © Whena question uses the word indicates, go back to the passage and find exactly what the pas- sage actually says. Eliminate (A) because it’s never stated that the mine was first established by the Egyptians, Choice (B) can be eliminated based on the information provided in the first sentence of the passage—the road begins aa climb so steep that the Péischen Pass seems a mild slope by comparison. Choice (C) is supported by lines 5-1] in paragraph one, so keep it. Line 7 says bleak stone building, but it’s never stated that it’s boring, so elirainate (D). Choice (C) is the best answer 26. Cross off the word shore and use the context in the sentence to come up with your own word as a replacement, A good replacement would be support since the catacombs must contend with the pressure af the ‘mountain and they wouldn't merge together. Choices (A), (B) and (D) are too close to the physical definition of shore, whereas buttress means to support. Choice (C) isthe best answer. 27. D___ The Steinberg mine is discussed in paragraph four, so use POE and look to see which answer choice is supported by the paragraph. There is no evidence that the same family owned the mine, so eliminate (A). Choice (B) is partially correct because lines 48-50 suggests that water instead of picks and shovels were used to ‘mine salt. However, we don't know if this is unique, so get rid of (B). While ic may have been isolated, that’s not the same as inaccessible, so eliminate (C). Choice (D) works because lines 44-47 state throughout the centuries, as cities and empires rose and fell the mine continued to produce salt. So, (D) works, 28. A The correct answer to the previous question is (1D). ‘The evidence for this answer is expressed in the first sentence of paragraph four, so therefore (A) is the best answer. 29. B__Thesecond-to-last paragraph opens with since she 1300s, this job had been performed by members of a small group of families, whereas the final paragraph contrasts this with the salt mine at Aleaussee was assaulted by the modern world. The next sentence then describes a seemingly endles line of Nasi oficers descended on the mine as guards. Chie (B) isthe best macch for this contrast. 30. € __ The answer to the previous question indicates that the passage contrasts local mining families and Nazi officers. Choice (A) describes several characteristics of the minors without setting up or explaining this contrast. Therefore choice (A) is incorrect. Choice (B) describes the arrival of the modern world in the mines, ‘without clarifying who exactly is arriving in the mines. Choice (B) can be eliminated. Choice (C) describes the aurival of Nazi soldiers, which does specifically support the contrast described in question 29, Choice (C) is che correct answet. Choice (D) describes the use of wooden racks, which doesn't support the previcus answer. Choice (D) is incorrect. 31. C Since this isthe first time that arthas been mentioned in this passage, then this indicates thae the sale mines may have an alternative purpose than the production of salt. Choice (C) is a good fit for this. Eliminate (A) since sculpting is never mentioned, as well as (B) since it’s never stated that the Nazis considered mining an art form. Choice (D) also doesn't work because there is no mention of a mining museum. TPM Edun Hodes UE ‘This passage Is excerpted from "Trouble at the Lab an article published by The Economist on October 19, 2013, “L see a train wreck looming” warned Daniel Kahneman, an eminent psychologist, in an open letter last year. The premonition concerned research on a Line phenomenon known as ‘priming’. Priming studies 5 suggest that decisions can be influenced by apparently irrelevant actions or events that took place just before the cusp of choice They have been a boom area in psychology over the past decade, and some of their insights have already made it out of the lab and into 10 the toolkits of policy wonks keen on “nudging” the populace. Dr: Kahneman and a growing number of his colleagues fear that a lot of this priming research is poorly founded. Over the past few years various 15 researchers have made systematic attempts to replicate some of the more widely cited priming experiments ‘Many of these replications have failed. In April, for instance, a paper in PLoS ONE, a journal, reported that nine separate experiments had not managed to 20 reproduce the results of a famous study from 1998 purporting to show that thinking about a professor before taking an intelligence test leads to a higher score than imagining a football hooligan. “The idea that the same experiments always get the same results, no matter who performs them, is one of the cornerstones af science’ claim to objective truth. Ifa systematic campaign of replication does not lead to the same results, then either the original research is flawed (as the replicators claim) or the replications 50 are (as many of the original researchers on priming. contend). Bither way, something is awry. It is tempting to see the priming fracas as an isolated case in an area of science—psychology—easily marginalized as soft and wayward, But irreproducibility 25 is much more widespread. A few years ago scientists at ‘Amgen, an American drug company, tried to replicate 53 studies that they considered landmarks in the basic science of cancer, often co-operating closely with the original researchers to ensure that their experimental 40 technique matched the one used first time round. According to a piece they wrote last year in Nature, a leading scientific journal, they were able to reproduce the original results in just six ‘Academic scientists readily acknowledge that 45 they often get things wrong. But they also hold fast to the idea that these errors get corrected over time as other scientists try to take the work further. Evidence that many more dodgy results are published than are subsequently corrected or withérawn calls that much- 50 vaunted capacity for self-correction into question. There are errors in a lot more of the scientific papers being, published, written about and acted on than anyone ‘would normally suppose, or like to think. Various factors contribute to the problem. Statistical 15 mistakes are widespread, The peer reviewers who evaluate papers before journals commit to publishing them are much worse at spotting mistakes than they or others appreciate. Professional pressure, competition and ambition push scientists to publish more quickly @ than would be wise. A career structure which lays great stress on publishing copious papers exacerbates all these problems. “There is no cost to getting things wrong” says Brian Nosek, a psychologist at the University of Virginia who has taken an interest in his discipline’ 65 persistent errors. “The cost is not getting them published” In testimony before Congress on March Sth Bruce Alberts, then the editor of Science, outlined what needs tobe done to bolster the credibility of the sciemtific 70 enterprise. Journals must do more to enforce standards, Checklists such as the one introduced by Nature should be adopted widely, to help guard against the most common research errors, Budding scientists must be taught technical skills, including statistics, and must 75 be imbued with skepticism towards their own results and those of others. Researchers ought to be judged on the basis of the quality, not the quantity, of their work, Funding agencies should encourage replications and lower the barriers to reporting serious efforts which 20 failed to reproduce a published result. Information about such failures ought to be attached to the original publications. ‘And scientists themselves, Dr Alberts insisted, “need to develop a value system where simply moving on 45 from ones mistakes without publicly acknowledging them severely damages, rather than protects, a scientific reputation” This will not be easy. But if science is to stay on its tracks, and be worthy of the trust so widely invested in i, it may be necessary. ‘As used in line 48, “dodgy” most nearly means A) quick B) unreliable. ©) putrid. D) exposed. Based on the information in the passage, it can be reasonably inferred that A) researchers may publish results that are not statistically sound, B) scientists are often forced to publish before their work is ready, ©) psychology experiments dontt produce substantiated facts. D) scientific researchers cat reproduce most oftheir ‘experiments. In discussing the phenomenon known as “priming”, the author of the passage suggests that A) itis. theory proven by systematic evidence. B)_itaffects how people vote. ©) itis not soundly based on repeatable experiments. D) itis influenced by irrelevant actions. ‘Which choice provides the best evidence for the answer to the previous question? A) Lines 4-7 ("Priming studies... . choice”) B) Lines 7-11 (“They have... . populace”) ©) Lines 12-14 (“Dr Kahneman ... founded”) D) Line 17 (“Many ... failed”) As used in line 61, “exacerbates” most nearly means A) causes. B) frustrates. C) diminishes. D) compounds. ‘The author's reference to the “cornerstones of science’s claim to objective truth” in line 26 primarily serves to A) debunk a flawed claim, B) acknowledge a potential consequence. C)_ underscore a tenet of scientific thought. D) correct an earlier assumption. ‘he tone of the passage is best described as A) perfunctory. B) cautionary. ©) ironical. D) inflammatory. aa a As used in line 32, “the priming fracas,” refers to A) the uproar caused by certain psychological studies. B)_alandmark cancer experimental technique. ©) asingle case that went awry. D) the results of several of the 53 studies done by ‘Amgen. 10 In line 53, what is the most likely reason the author says, “or like to think”? A) to-clarify that not everyone agrees withthe results of experiments. B) to evaluate the concern for peer review of scientific journals. ©) to rationalize why researchers make mistakes and still publish their work. D) to stress that errors persist because scientists cling. to biased beliefs, orPREwestnn aes UC | SY EEE Which choice provides the best evidence for the In the sixth paragraph, Dr. Brian Nosek draws a answer to the previous question? distinction between A) Lines 24-26 (“The idea .. . truth”) : A) individual persistence and academic considerations. B) Limes 44-45 (‘Academic ... wrong”) (©) Lines 45-47 ("But they ... further”) D) Lines 55-58 ("The peer ... appreciate”) B) scientific results and career ramifications. ©) institutional competition and financial loss. D) professional ambition and personal desire. 12 | erpaearcontP Hoaees We THIS PAGE IS LEFT INTENTIONALLY BLANK. Answers and Explanations 32, B “The author says scientists make mistakes, but they assume that the errors will be corrected over time, However, he says, more of these dodgy results are published than are subsequently corrected or withdrawn. This means that the results are incorrect and unreliable. Choice (B) is thus the correct answer. The word dodgy, Choice (A), sounds like dodging or moving quickly, but this is a trap answer. The results are not rotting, so (C) is incor rect. Choice (D) is incorrect, since the errors in the results are not revealed or exposed. 33. A The passage states that irreproducibility is widespread, buc not that researchers can’t reproduce most of their experiments, so (D) is not the correct answer. The passage says psychology is easily marginalized for being too sofe, but (C)is too extreme, Scientists are pressured to publish, but the passage doesn say that they do it before their work is ready, so (B) is incorrect. Choice (A) can be backed up by the passage when it says, There are errors in alot more of the scientific papers being published, written about and acted on than anyone would normally suppose, or like to think. 34. C The passage states that priming has come under suspicion duc to the fact that people have not been able to reproduce the results of prior experiments. Choice (C) is thus the correct answer. Priming has not been proven by systematic evidence, so (A) is incorrect. It was believed to have been able to influence votes, but that has been called inco question so (B) is incorrect. Choice (D) is wrong because it uses words from the passage, but doesn't answer the question, 35. D__ The passage says, Many of these replications have failed, This supports the answer stating that priming is nor soundly based on repeatable experiments. Thus, (D) is the correct answer, and you can eliminate (A), (8), and (©). 36. D___ The passage states that various factors contribute to the problem of errors in rescarch studies, then describes them. The passage adds that the academic career structure pushes people to publish papers, thus making the situation worse. Compounds means to intensify the negative aspects, so (D) is correct. The pressure to publish isn'e the lone cause, so (A) is wrong, The word frustrates in this context would mean to prevent the problem and diminishes means to reduce, so (B) and (C) are incorrect. 37. © Choice (C) is correct because the author is pointing out that this issue is especially problematic since it goes against a2 cept integral to scientific thought. The author is not trying to disprove a claim, so (A) is incorrect, nor is the author attempting to correct any ideas from earlier in the passage, so (D) is incorrect. ‘The potential consequences are not described here, so (B) is wrong 54 | ctrnceuctont Hoge ic 38. B_—Theauthor’s last sentence warns, But if science isto stay on it tracks, and be worthy ofthe trust so widely invested in it, it may be necessary, so the tone is cautionary, (B). The author clearly puta lot of efore into this research, 10 its not (A). Some of the results may be different than we expect, but the author doesn write with 4 tone that is ironic, making (C) wrong. Again, some of the information in che passage could arouse feelings of anger, but inflammatory, (D) is too extreme. 39. A The priming fracas refers to the potential “train wreck? that Dr. Daniel Kahneman refers to in the opening of the article. The issue over priming research led co many other problems with the reliability of published research in general So, its not a single case, (C), or single technique, (B), or just the seudies done by ‘Amgen, (D), buta large-scale issue with ramifications for many studies, Thus, (A) is correct. 40. Cc ‘The reason the author says or like to think is to make it clear that the current situation is not only contrary to common perceptions, but also something tha people have convinced themselves to believe. In other words, its a rationalization, which makes (C) correct. Choices (A) and (D) might be true, but they don't explain the use of the phrase. ‘The phrase isn used to judge or evaluate peer review of scientific journals, making (B) incorrect. 41. C The passage says, Bur they alo hold fast to the idea that these errors get corrected overtime as other seientist ry to sake the work further, which supports the author’s use of “like to think” to suggest that researchers ae willing to publish flawed research due to faulty rationalizations. Thus, (C) is the correct answer, and you can climinate (A), (B), and (D). 42, B___Dr, Brian Nosek says that there isa price for not publishing, which is losing a job or funding, but there are no ramifications for publi researcher makes for career reasons Versus 8 academia, so (A) is incorrect. Choices (C) and (D) show ewo things chat are more similar than distinct, according to the passage, so they arc incorrect. formation that is wrong. The difference is becween the choices a truth. Thus, (B) is correct. According to the passage, persis- tence is not rewarde 2» 2s 20 6 ‘The following passages is excerpted from the article "Wales: ‘Awalkalong Offa's Dyke Path with Bil Bryson” by John Flinn, published in 2012 In the Los Angeles Times. “The pathway, which winds 177 miles from the River Severn in the south to the Irish Sea in the north, loosely follows an earthen “dyke” as the Brits spell it, “dike” as Americans do—a dirt wall and ditch—built in the late eighth century by the Anglo-Saxon King Offa of Mercia to keep out the Wash. Today the dike, the footpath and the modern border between England and Wales all traverse roughly the same ground, crisscrossing each other like strands in a braid. (Our route traced a hedge-lined country lane and wandered past the tottering stone barn and outbuildings ofa small farm, where the farmer, standing in mud- encrusted Wellington boots next to an ancient, mud- encrusted Range Rover, nodded a hello, At the edge of his pasture we scampered over a stile, alittle stepladder to help walkers surmount fences and hedges. ‘Across large swaths of the bucolic British ‘countryside, thereis no such thing as trespassing, The British staunchly protect and defend their right to amble wherever their feet may take them—even across fenced-in pastures. It was enshrined in a national Right to Roam law in 2000, but the impulse dates back. almost two centuries, when poet William Wordsworth, described the Lake District as “a sort of national property, in which every man has aright and interest ‘who has an eye to perceive and a heart to enjoy” ‘Over the next five days, wandering back and forth across the Anglo-Welsh border—which was rarely marked—we walked alongside Offas Dyke for long stretches and even atop it for a short spell. A weathered mound of dirt 4 to 8 feet high, its not nearly as impressive as the Roman-built Hadrian's Wall, which stretches imperially across northern England. King Offa appears to have constructed his dike more as a line of demarcation—a political statement—than as an actual defensive barrier. But woe to anyone caught ‘on the wrong side of it in Offas day. According to George Borrow'’s “Wild Wales,” “it was customary for the English to cutoff the ears of every Welshman who ‘was found to the cast of the dike, and for the Welsh to thang every Englisaman whom they found to the west of it” Its been called “England’s greatest surviving Anglo- Saxon monument” Its certainly the longest. One afternoon, as we left the dike and descended a 45 steep hillside above the village of Llanfair Waterdine, we heard a bleating commotion and looked down to see roiling sea of wool surging toward us. Llewelyn Morgan, a shepherd straight out of central casting, was moving his flock to a higher pasture. Actually, his dog Sandy, 59 an energetic and no-nonsense taskmaster, was doing all the heavy lifting. Morgan flashed hand signals and occasionally whistled, and Sandy raced back and forth around the perimeter of the flock, herding them into a tight, orderly bunch. Once a couple of sheep made a 55. mad dash to escape, but Sandy chased them down and coerced them back to the flock, nipping at their heels. Elsewhere in Britain, I've attended formal sheep dog trials, but theres nothing like seeing the real thing in action, 60 “Icould have four men helping me?’ said Morgan, “but they couldn't do what Sandy does.” As we chatted with Morgan, I tried to burn the scene into my memory. For this too was another tableau of British country life that could be gone a generation from now, The next day 6 we came across a younger shepherd herding his flock. He had no dog, and he was riding an all-terrain vehicle “Thats what's so wonderful cbout walking,” Bryson said later. “If we were passing by at 60 mph on the motorway, we wouldn't have seen any of this” ‘The primary purpose of the passage as a whole is to ‘A) bemoan the environmental damage done by motorways. B) offer a comprehensive description of the history of Offs Dyke. ©) relate the picturesque details of journey across a changing countryside. D) argue that the bucolic English countryside should resist modernity. In discussing Offa's Dyke, the author of the passage suggests that A) itis bigger than Hadrian's Wall B) it was built to keep the Welsh out of England. ©) itis the official border between Wales and England. D) it isa type of hedgerow. Which choice provides the best evidence for the answer to the previous question? A) Limes 1-6 ("The pathway, ... Welsh’) B) Lines 6-9 (*Today.... braid”) ©) Lines 33-36 ("King Offa... barrier”) D) Lines 36-37 ("But woe... . day”) 1 | ‘As used in line 15, “scampered” most nearly means A) wandered. B) stumbled ©) ambled. D) scuttled. As used in line 26, “aheart to enjoy,” refers to A) anational park for all to appreciate 1B) a capacity to commune with nature ©) abealthy constitution D) a love ofall things beautiful Lis] ‘As used in line 35, “demarcation” most nearly means A) distinction. B) expulsion, ©) fortification. D) enclosure Based on the informetion in the passage, it can be reasonably inferred that AA) the British protect and defend their property staunchly. B) the Right to Roam law was opposed for more than two centuries. C)_ trespassing is never against the law in Great Britain, D) itis possible to meander on private property in the English countryside. ‘Which choice provides the best evidence for the answer to the previous question? A) Lines 6-9 (“Today .. braid”) B) Lines 18-21 (“The.... pastures”) ©) Lines 33-36 ("King .. barrier”) D) Lines 37-40 (‘According ... dike”) ‘The “sea of woo!” referred to in line 47 is “roiling” because ‘A) the waters of the River Severn were churning. B) the wool was white like the caps of ocean waves. ©) the large herd of sheep moved as a swirling mass. D) the sheep kicked up dust as they moved, In the sixth paragraph, the passage mentions a distinction between ‘A)_ Llewelyn Morgan and the all-terrain vehicle B) the past and the future ©) Sandy and the young shepherd D) the sheepdog and human helpers {In line 62, what is the most likely reason the author says, “I tried to burn the scene into my memory”? A). He fears he wor't see these sorts of images in the future. B) He wants to be able to write about it later. C) Its unlike anything he’ ever seen before. D) The sheepherder doesn't want his picture taken for fre. eTPRehocatonP tons. UC | Answers and Explanations 11. © Choice (A) is too specific for a central idea question, while (B) is too broad. ‘The author does seem to appreciate the English countryside as it is, but the passage doesn’t strongly argue against the changes, making (D) wrong. Thus, (C) is correct, since the passage as a whole relates a trip and what the author saw. 12. B Choice (A) is incorrect, since the passage says Hadrian's Wall is more impressive and then de- scribes Offals Dyke as only four to eight feet high. Choice (B) is correct, since the passage says King Offa buile the wall co keep out the Welsh. ‘The passage describes the pathway, the dike, and the modern border of Wales and England as crisscrossing each other, so the dike cant be the border, which makes (C) incorrect. Choice (D) is wrong because it uses a word from the passage that does not refer to or describe the dike. 13. A The passage says “built in the late eighth century by the Anglo-Saxon King Offa of Mercia to keep out the Welsh.” Thus, (A) is the correct answer, and you can eliminate (B), (C), and (D). 14. D__ The walkers climbed quickly over the fence, so (A) and (C) are wrong, since those words de- scribe walking in a leisurely way. There's nothing to indicate that the walkers almost fll, so (B) is incorrect. ‘Thus (D) is correct. 6 BO in of nature, so (B) is correct. Wordsworth says the lands are for all, who can, to appreciate, but this phrase is not referring to a national park. Thus, (A) is incorrect. Choice (C) refers to an actual healthy heart, which is wrong, and (D) is extreme, sntext of the sentence, a heart to enjoy specifically refers to the ability to enjoy the beauty 16. A Thedike was a line differentiating England from Wales, so (A) is the correct answer, The Welsh did not Expel the English found on the West side of it, they hanged them, so (B) is incorrect, The passage states that the dike was not a defensive barrier, so (C) is incorrect. Choice (D) is incorrect, since the dike doesn't fully seal off Wales. 17. D___ The passage states that the English have historically allowed people to wander theit lands and a law passed in 2000 officially made it legal, so (D) is the correct answer. The passage never says that the Right to Roam law was opposed, so (B) is wrong. Choice (A) is incorrect because the passage discusses how the British defend their right to wander, nor their property. Choice (C) is too extreme, since the passage doesn’t say it’s never against the law to trespass. 18. B__The answer co the previous question indicates that the English have the right to wander the countryside. Choice (A) just describes the country side, so it is incorrect. Choice (B) discusses the British right to ‘wander so it supports the previous answer. Both (C) and (D) talk about port the previous answer. ical examples, and so do not sup- ©rPaehesio dee LC 19. © Choice (C) is correct, since roiling means to move in a swirling, turbulent manner, which is how the sheep were moving. Choice (A) is incorrect, since it doesn't address the fact that it’s a herd of sheep. Choice (B) docsnit address the use of roiling and (D) is incorrect because it brings in the idea of dust when “roiling” is referring to the movement ite, 20. D__ The passage implies that Morgan and the young shepherd are different, but ao clear distinction is made between Morgan and the vehicle, (A) or Sandy the dog and the young shepherd, {C). Thus, they are incorrect. Choice (B) sounds good, but Morgan represents the present and the young sheepherder the possible future. The only clear distinction drawn is between Sandy the dog and human helpers, sincz Morgan says that four men couldn't do what Sandy does, making (D) correct, 21, A Theauthor says chat shepherding is another part of English country life chat could be gone in 4 generation, Thus, (A) is the correct answer. Choices (B) and (©) are incorrect even though the author may have wanted ro remember the scene to write the article and he may not have seen anything like it, but the purpose of writing chat ine at that point is so the author can clarify how he feels about something that may disappear in the future. Choice (D) is incorrect since there's no reference to how the shepherd fele about having his picture taken. Traitor olin. | 8 ‘The passage below is adapted from an article discussing ‘minor Elizabethan dramatists. t focuses on the works Thomas Heywood and Thoma: Middleton, two influential playwrights ‘of the early seventeerth century. ‘Thomas Heywood, of whom little is known, was one of the most prolific writers the world has ever seen. In 1598 he became an actor, or, as Henslowe, who Line employed him, phrases it, “came and hired himself to 5 me as a covenanted servant for two years.” The date of his first published drama is 1601; that of his last published work, a “General History of Women,’ is 1657. As early as 1633 he represents himself as having had an “entire hang, or at least a main finger” in two 10 hundred and twenty plays, of which only twenty-three were printed, “Itis true.” he says, “that my plays are not exposed to the world in volumes, as others are: one reason is that many of them, by shifting and change of companies, have been negligently lost; others of 15 them are still retaired in the hands of some actors, who think it against their peculiar profit to have them come in print; it was also never any great ambition in ‘me to be in this kird voluminously read.” It was said of him, by a contemporary, that he “not only acted 20 every day, but also obliged himself to write a sheet every day for several years; but many of his plays being ‘composed loosely in taverns, occasions them to be so mean.” Besides his labors asa playwright, he worked as translator, versifier. and general maker of books. Late in 25 life he conceived the design of writing the lives ofall the poets of the world, including his contemporaries. Had this project been carried out, we should have known, something about the external life of Shakespeare; for Heywood must have carried in his brain many of those 40 facts which we of this age are most curious to know, Heywood’ best plays evince large observation, considerable dramatic skill, a sweet and humane spirit, and an easy command of language. His style, indeed, is singularly simple, pure, clear, and straightforward; 235 but it conveys the impression of a mind so diffused as almost to be characterless, and incapable of flashing its thoughts through the images of imaginative passion. He is more prosaic, closer to ordinary life and character, than his contemporaries. 40 With less fluency of diction, ess skill in fastening the reader's interest to his fable, harsher in versification, and generally clumsier in construction, the best plays of Thomas Middleton are still superior to Heywood’s in force of imagination, depth of passion, and fullness 45 of matter. It must, however, be admitted that the orPaesseatonP Haine sentiments which direct his powers are not so fine as Heywood’. He depresses the rind, rather than invigorates it. The eye he cast on human life was not the eye of a sympathizing poet, but rather that of a 50 sagacious cynic. His observation, though sharp, close, and vigilant, is somewhat ironic and unfeeling. His penetrating, incisive intellect cuts its way to the heart of a character as with a knife; and ifhe lays bare its throbs of guilt and weakness, and lets you into the secrets of its 55. organization, he conceives his whole work is performed, ‘This criticism applies even to his tragedy of “Women beware Women,’ a drama which shows a deep study of the sources of human frailty, considerable skill in exhibiting the passions in their consecutive, if not in 60 their conflicting action, and a firm hold upon characters but it lacks pathos, tenderness, and humanity; its power is out ofall proportion to its gemvalitys the characters, while they stand definitely out to the eye, are seen through no visionary medium of sentiment and fancy 6 There is, indeed, no atmosphere to Middleton's mind; and the hard, bald caustic peculiarity of his ‘genius, which is unpleasingly felt in reading any one of his plays, becomes a source of painful weariness as we plod doggedly through his works. This is most 70 powerfully felt in his tragedy of “The Changeling” at once the most oppressive and impressive effort of his genius. The character of De Flores in this play has in a strangeness such as is hardly paralleled in the whole range of the Elizabethan drama. The passions of this 75 brute imp are not human. They are such as might be conceived of as springing from the union of animal with fiendish impulses, in a nature which knew no law outside ofits own lust, and was as incapable ofa scruple as of a sympathy. 1 22 | ‘The primary purpose of the passage is to ‘A)_ compare the personal lives of two playwrights. B) examine and contrast the works of two individuals ©) advocate a method of evaluating dramatic works. D) describe the role that dramatists have played in history. According to the information in the passage, the author most likely woald agree that Heywood ‘A) could have contrizuted more to our knowledge of influential seventeenth century writers than he actually did B) was more involved in professions other than playwriting than many authorities today believe. (©) was an actor in more than two hundred plays, although only slightly more than twenty became popular productions. D) would have been the most talented playwright of his day had he possessed more imagination and passion. ich choice provides the best evidence for the answer to the previous question? A) Lines 3-11 ("in 1:98... printed”) B) Lines 18-24 (“It was said... books") ©) Lines 24-30 (“Late .. know”) D) Lines 33-39 (*his style... contemporaries”) ‘The author's reaction “0 Middleton is best described as amixof ‘A)_ admiration for his ingenuity but criticism for his, absence of warmth. B) disgust for is style but appreciation for his displays of tenderness. C) bewildered by his use of diction but curious about his sentiments, D) apathy toward his ability to dishearten readers but dislike of his coldness. According to the information in the passage, one primary difference be:ween Heywood and Middleton is that Heywood ‘A) displayed a more cynical attitude toward humanity. B) had fewer of his works published. ©) was a more disciplined writer. D) showed less powe-ful creativity in his writing, Which choice provides the best evidence for the answer to the previous question? A) Lines 8-11 ("As early as BL ©) Lines 40-45 (“With les. D) Lines 48-50 (“The eye - primed”) es 18-23 (“It was said ... mean”) matter”) «-eynic’) Heywood claims that all of the following are reasons that many of his plays were not published EXCEPT A)_ many of his plays accidentally went missing. B) Heywood lacked a strong desire for popularity ©) anumber of his plays feature mean characters. D) some individuals kept the plays to prevent their publication a Asused in line 25, “design” most nearly means A) draft B)_ pattern ©) biography. D) intention. 130 | Asused in line 46, “fine” most nearly means A) dainty. B) admirable ©) skill D) exact. The information in lines 51-55 serves primarily to A) reveal the primary method by which a dramatist gained inspiration for his plays. B) analyze the way a playwright shared his innermost emotions with his audiences. ©) argue that dramatists must display keen intelligence in order to attract audiences. D) demonstrate the piercing style of an important literary figure by using an illustration, orPReavason Pegs LC | 6 Answers and Explanations 22, B__The ight answer is (B), The passage is primarily about the difference between the works of the ‘wo writers, not their personal lives, which is why (A) is noc the right answer. The primary purpose is not to advocate a method of evaluating plays so the answer is not (C). The answer isn’t (D) because both writers’ works are analyzed within their time periods, not within a greater historical context. 23. A The sight answer is (A). The author writes that late in life Heywood conceived the design of writ- ing the lives of all the poets of the world, including his contemporaries. Since the project was not carried out we do not have this knowledge, but it would have been a great contribution had he completed the project. Choice (B) is not the right answer since we have no reason to think he was involved in more projects that we don't know abour. ‘The answer is not (C) because while we know he was involved in 200 plays and we know he acted ia some, we don't know that he acted in more than 200 plays. Choice (D) isa matter of opinion not supported in the passage 24. © Theanswer is (C). Only (C) addresses Heywood’s contributing knowledge of other seventeenth- century playwrights, Choices (A), (B), and (D) discuss his output as a writer, his personal connectiens, and his style. But only (C) addresses his possible contribution to the history of the period in which he worked. 25. A The author describes Middleton as a writer possessing great incisiveness, creativity, and inven- tiveness. In lines 72-74 the author praises one of Middleton's characters as having a strangeness such as is hardly paralleled in the whole range of Elizabethan drama. However, the author also points out that Middleton's works often lack pathos, tenderness, and humanity. This mixed reaction is best summarized in (A). Choice (B) can be liminated because although the author was not overly impressed with Middleton’ style, disgust is too extreme. Additionally, the author does not believe that Middleton displayed much in the way of tenderness. Choice (C) can be eliminated, because the author is never bewildered by any aspect of Middleton's writing, Apathy and dislike are both too extreme to describe the author's view of Middleton's work, so (D) can be eliminated. 26. D__Theanswer is (D). In ines 42-45, the author states that Middleton's works are superior to Hey- wood’ in force of imagination, depth of passion, and fullness of matter. Choice (A) is directly contradicted by line 32, which stares that Heywood possessed a sweet and humane spirit, Choices (B) and (©) are not supported in the passage. 27. € Asis noted in the explanation for question 26, Middleton's plays are more imaginative and passionate than those of Heywood. Only (C) addresses this directly. Choices (A) and (B) focus on aspects of Heywood's writing and career, and answer (D) makes an observation on Middlevon’s character. Only (C) focuses on the differences between the two writers. 28. Lines 12-18 indicate that a number of Heywood's plays were negligently lost So (A) is not cor- rect, These lines also indicate that some actors prevented them from going to print, so (D) is wong. Heywood also admits iewas no great ambition..to be read. So (B) is wrong. Choice (C) is right because it never mentioned. in the passage. 29. D__Theanswer to this (D). Ifyou fill in the blank with your own definition, you should get some- thing like ides, When you look through the answer choices, idea is closest to (D), intention, All the others are ‘ap answers. Choice (A) would seem to relate toa writing project, but we are looking for the idea ofthe project, not the first draft of it. Choice (B) doesn't make sense in the contexc of che sentence, And while the final product ‘might be a biography, Heywood is still atthe beginning phase of the project. So, (C) can be diminated. 30. B___Inllines 45-50, the author says chat Middleton’ sentiments are not so fine as Heywood, The lines farther state that Middleton depresses the mind, makes ironic and unfeeling observations, and views the world not 38 a.ympathetic poet but asa sagacious cynic, Taking all of this together, it can be surmised that the author finds Middleton's sentiments less appealing and beneficial than Heywood’. Choice (B) best matches this meaning, Although the author mentions that he finds Middleton's writing less skilful and exact than Heywood’, these qualities are not discussed in these lines. Therefore (C) and (D) can be eliminated. Choice (A) isa trap answer, as it isa more common definition for fine, bu ie doesnt make sense in context. 31. The correct answer is (D). ‘The phrase incisive intellect... fewt)..its way to the heart ofa character as with a knife describes the power of Middleton's writing. Choice (D) best captures this idea, The author never mentions how Middleton got his inspiration, so (A) can be eliminated, ‘The passage doesn’t give us enough infor- ‘mation about Middleton's emotional life to prove (B), so it can be eliminated. Choice (C) discusses dhammatistsin general rather than the single dramatist under discussion. orPReavannt Heras | 6B ‘The following is an excerpt from the 1913 autobiography of ‘Theodore Roosevelt J, who served as the 26” President of the United States, In the passage below he discusses childhood memories of his father. My father, Theodore Roosevelt, was the best man lever knew. He combined strength and courage with gentleness, tenderness, and great unselfishness, He Line would not tolerate in us children selfishness or cruelty, 5 idleness, cowardice, or untruthfulness, As we grew older he made us understand that the same standard of clean living was demanded for the boys as for the girls; that what was wrong in @ woman could not be right in aman, He combined insistence on discipline with 10 great love and patience, and the most understanding sympathy and consideration. He never physically punished me but once, but he was the only man of ‘whom I was ever really afraid. I do not mean that it was, a wrong fear, for he was entirely just, and we children 15 adored him. We used to wait in the library in the evening until we could hear his key rattling in the latch of the front hall, and then rush out to greet him; and we would troop into his room while he was dressing, to stay there as long as we were permitted, eagerly examining 20 anything which came out of his pockets which could be regarded as an attractive novelty. Every child has fixed in his memory various details which strike it as of grave importance. The trinkets he used to keep in a little box on his dressing-table we children always used to speak 25 of as “treasures.” The word, and some of the trinkets ‘themselves, passed on to the next generation. My own children, when strall, used to troop into my room while Iwas dressing, and the gradually accumulating trinkets, in the “ditty-box”—the gift of an enlisted man in the 30 navy—always excited rapturous joy. On occasions of solemn festivity each child would receive a trinket for his or her “very own.” My children, by the way, enjoyed one pleasure I do not remember enjoying myself. When T came back from riding, the child who brought the 35 bootjack would itself promptly get into the boots, and clump up and down the room with a delightful feeling ofkinship with Jack of the seven-league strides. The punishing incident I have referred to happened ‘when I was four years old. I bit my elder sister's arm, 1 40 do not remember biting her arm, but I do remember running down to the yard, perfectly conscious that I had committed a crime. From the yard I went into the kitchen, got some dough from the cook, and crawled under the kitchen table. In a minute or two my father 45 entered from the yard and asked where I was. The ‘warm-hearted Irish cook had a characteristic contempt for “informers, but although she said nothing she ‘compromised between informing and her conscience by casting @ look under the table. My father immediately 50 dropped on all fours and darted for me. I feebly heaved the dough at him, and, having the advantage of him because I could stand up under the table, gota fair start for the stairs, but was caught halfway up them, The punishment that ensued fitted the crime, and I hope— 45 and believe—that it did me goo. Tnever knew anyone who got greater joy out ofliving than did my father, or anyone who more whole-heartedly performed every dutty; and no one whom I have ever met approached his combination 0 of enjoyment of life and performance of duty. He and my mother were given to a hospitality that at that time ‘was associated more commonly with southern than northern households; and, especially in their later years, when they had moved up town, in the neighborhood of ‘65 Central Park, they kept a charming, open house. In the passage, Roosevelt primarily attempts to A) provide a character sketch of an important figure in hislife B) portray an incident in which he behaved poorly. ©) reflect on the small things in life which children enjoy. D) consider the causes of his fear of displeasing his father. 12] Roosevelt’s attitude towards his father could best be described as a mixture of A) fear and discomfort. B) admiration and respect. ©) amusement and disdain. D) love and envy.

You might also like